SlideShare a Scribd company logo
1 of 53
Download to read offline
1
MAXIMA & MINIMA
A SHORT NOTES
Arun Umrao
https://sites.google.com/view/arunumrao
DRAFT COPY - GPL LICENSING
2 Maxima & Minima
Contents
1 Extrema 3
1.1 Maxima & Minima . . . . . . . . . . . . . . . . . . . . . . . . 3
1.1.1 Maximum & Minimum Function . . . . . . . . . . . . . 3
1.2 Some Definitions . . . . . . . . . . . . . . . . . . . . . . . . . 4
1.2.1 Maxima & Minima Points . . . . . . . . . . . . . . . . 5
1.2.2 Critical Points . . . . . . . . . . . . . . . . . . . . . . . 5
1.2.3 Stationary Points . . . . . . . . . . . . . . . . . . . . . 6
1.2.4 Point of Inflection . . . . . . . . . . . . . . . . . . . . . 6
1.2.5 Extreme Value Theorem . . . . . . . . . . . . . . . . . 8
1.2.6 Global Minimum & Global Maximum . . . . . . . . . . 8
1.2.7 Local Minimum & Local Maximum . . . . . . . . . . . 9
1.2.8 Saddle Point . . . . . . . . . . . . . . . . . . . . . . . . 10
1.3 Extremum Test . . . . . . . . . . . . . . . . . . . . . . . . . . 11
1.4 Method of Finding Extremum . . . . . . . . . . . . . . . . . . 11
1.4.1 First Derivative Test . . . . . . . . . . . . . . . . . . . 11
1.4.2 Second Derivative Test . . . . . . . . . . . . . . . . . . 12
1.4.3 Why Second Derivative Required . . . . . . . . . . . . 13
1.5 Extremum in Two-Dimensional Function . . . . . . . . . . . . 15
1.6 Applied Problems . . . . . . . . . . . . . . . . . . . . . . . . . 16
1.6.1 Hypothetical Extrema . . . . . . . . . . . . . . . . . . 29
1.1. MAXIMA & MINIMA 3
1Maxima & Minima
1.1 Maxima & Minima
Maxima and Minima are points where function reaches its maximum and
minimum value respectively. These points are also called extrema. Extrema
are of two types: (i) Global extrema (Absolute extrema) and (ii) Local ex-
trema (Relative extrema). Global extrema are those points whose value is
maximum or minimum on the entire range of the function. Local extrema
are the largest or smallest values of the function in the immediate vicinity.
At extrema, the slope of a function in graph is necessarily zero. Extrema are
also called stationary points or turning points.
1.1.1 Maximum & Minimum Function
Certain functions have maximum and minimum values at their certain points.
At other points function value oscillates between these two extreme values.
Illustrated Example Assume sine function y = a sin(x). The domain of
argument of sine varies from 0 to 2nπ. Range of sine function lies between
-1 to +1 irrespective of value of argument.
For Maximum y y shall be maximum if sin(x) is maximum, i.e. sin(x) = 1
or x = π/2. Now the y is y = a.
For Minimum y y shall be minimum if sin(x) is minimum, i.e. sin(x) =
−1 or x = −π/2. Now the y is y = −a.
1 2 3 4 5 6
x
y
a
−a
4 Maxima & Minima
Solved Problem 1.1 Find the maximum and minimum value of y =
a sin2
(x) + b cos2
(x) and a > b.
Solution The given trigonometric function is y = a sin2
(x)+b cos2
(x) and
it is periodic function. General value of arguments of sine and cosine varies
from 0 to 2nπ. Range of sine and cosine lies between -1 to +1. Hence y shall
continuously changes between its maximum and minimum values. Changing
the function for one trigonometric form
y = a sin2
(x) + b 1 − sin2
(x)

Simplifying it for sine
y = b + (a − b) sin2
(x)
For Maximum y y shall be maximum if sin2
(x) is maximum, i.e. sin2
(x) =
1 or sin(x) = ±1. Here taking only +1, x = π/2. Now the y is
y = b + (a − b) = a
For Minimum y y shall be minimum if sin2
(x) is minimum, i.e. sin2
(x) =
−1 or sin(x) = ±
√
−1. It is imaginary value for sine, hence not acceptable.
So for only acceptable values sin2
(x) = 0, i.e. x = 0. Now the y is
y = b + (a − b) × 0 = b
1 2 3
x
y
a
b
x
y
a
b
1.2 Some Definitions
Following definitions are useful in finding the maximum or minimum value
of a function.
1.2. SOME DEFINITIONS 5
1.2.1 Maxima  Minima Points
Maxima and minima points on combining known as extrema points. At
extrema points curves changes it direction. There may one or more than one
extrema points for a given function.
x
y
x = ay2
(0, 0)
b
(a)
x
y
x = a − y2
(0, 2)
b
(b)
Figure 1.1: Function plot.
From figure 1.1(1) point (0, 0) is the point at deepest position. This
point is known as the minima point in the given range of the function f(x).
Similarly from figure 1.1(2), point (0, 2) is the point at maximum height. It
is known as maxima point for given range of the function f(x).
1.2.2 Critical Points
A single variable function f(x) has critical points at all points x0 where
f
′
(x0) = 0
or f(x) is not differentiable at the given point.
x
f(x)
f(x) = 2x2
(0, 0)
b
f
′
(x) = 0
(a)
x
f(x)
f(x) = 4 − x2
(0, 2)
b
f
′
(x) = 0
(b)
Figure 1.2:
6 Maxima  Minima
In figure 1.2(1) the tangent at lowest depth point is parallel to the positive
x axis. Similarly in figure 1.2(2) tangent at maximum height of the curve is
parallel to the positive x axis. The points at which tangent is parallel to the x
axis are known as critical points or stationary points. At critical points slope
of tangent f
′
(x) is zero. Multi-variable function f(x, y) has critical points
where either the gradient df(x, y) is zero or partial derivative of function is
not defined at that point, ie ∂f(x, y)/∂x or ∂f(x, y)/∂y is undefined.
1.2.3 Stationary Points
Points at which the derivative of a function f(x) vanishes. For example, at
point x0 if first derivative of function is zero, i.e.
f
′
(x0) = 0
Then point x0 is known as stationary point. This stationary point may be
minimum, maximum or inflection point. Value of function at stationary point
is called stationary value.
Solved Problem 1.2 Find the stationary points of function f(x) = 2x − x2
.
Solution The stationary points of the function f(x) is given by f
′
(x) = 0,
i.e.
f
′
(x) = 2 − 2x = 0 ⇒ x = 1
The stationary point of the given function is x = 1.
1.2.4 Point of Inflection
It is a point on a curve at which the curvature or concavity changes sign from
plus to minus or vice-versa. At reflection point, curve changes from being
concave (concave downward) to convex (concave upward), or vice versa. If
f(x) is k times continuously differentiable in a certain neighbourhood of a
point x with k odd and k ≥ 3, while f(n)
(x0) = 0 for n = 2, . . ., k1 and
f(n)
(x0) 6= 0 then f(x) has a point of inflection at x = x0.
1.2. SOME DEFINITIONS 7
1
−1
1
−1
x
f(x)
b
f(x) = x3
f
′
(x) = 3x2
f
′′
(x) = 6x
Above graph is drawn for f(x) = x3
. The point x at which slope of curve
is parallel to horizontal axis, is found by using
f
′
(x) = 0
It gives x = 0. Now again derivating f
′
(x), we get
f
′
(x) = 6x
f
′
(x) is zero at x = 0, hence x = 0 is point of inflection of given function
f(x). In the following figure, function g(x) = x3
− 3x2
+ 2x is plotted. Its
derivative is g
′
(x) = 3x2
− 6x + 2.
1
−1
1 2
−1
x
g(x)
b
g(x)
g
′
(x)
g
′′
(x)
The point of inflection shall be where, there g
′′
(x) = 0. So,
g
′
(x) = 0 = 6x − 6
It gives, x = 1. This is inflection point for the given function g(x). Note that,
inflection point may or may not be a root point, i.e. zeros of the function.
For example, take
y = x3
− 3x2
− 144x + 432
8 Maxima  Minima
Its roots are x = −12, x = 3 and x = 12. The point of inflection will be
found by taking y
′′
= 0. So,
6x − 6 = 0 ⇒ x = 1
which is not root of the given function y.
1.2.5 Extreme Value Theorem
If a function h(x) is continuous on a closed interval [p, q], then h(x) has both
maximum and minimum on closed interval [p, q]. If h(x) has an extremum
on an open interval (p, q) then extremum occurs at a critical point.
1.2.6 Global Minimum  Global Maximum
The overall largest value of a set, function, etc., over its entire range is called
global maximum. While smallest value of a set, function, etc., over its entire
range is called global minimum. It is impossible to construct an algorithm
that will find a global maximum or global minimum for an arbitrary function.
We can check whether function has global extrema or not by just putting
x → ±∞. For example,
y = x2
− 2x + 4
global maxima as
lim
x→−∞
y = (−∞)2
− 2 × −∞ + 4 = ∞
and
lim
x→+∞
y = ∞(∞ − 2) + 4 = ∞
Thus function is not converging when x → ±∞. Using these tests, we can
expect local extrema.
1. When limx→−∞ y = limx→+∞ y = ±∞, i.e. both have same values
with then function may have odd numbers of extrema. This happens when
highest degree of function is even.
1.2. SOME DEFINITIONS 9
x
y
x → −∞ x → ∞
x
y
x → −∞ x → ∞
2. When limx→−∞ y = −∞ and limx→+∞ y = +∞ or vice-versa then
function may have even numbers of extrema. This happens when highest
degree of function is odd.
x
y
x → −∞ x → ∞
x
y
x → −∞ x → ∞
1.2.7 Local Minimum  Local Maximum
It is also known as relative minimum and relative maximum. As-
sume a function f(x), whose successive derivatives at point x0 are zero. i.e.
f
′
(x0) = 0, f
′′
(x0) = 0, . . . . . . , f(n)
(x0) = 0
But
f(n+1)
(x0) 6= 0
Now, f(x) has a local maximum at x0 if n is odd and f(n+1)
(x0)  0. Sim-
ilarly, f(x) is local minima at x0 if n is odd and f(n+1)
(x0)  0. The (n)th
derivative of function is zero at point x0 but (n)th
derivative of function is
not zero at point x0 hence it gives the maximum and minimum point. To
find the saddle point we have to find the maximum derivatives of function
that gives zero value at point x0 and also check is n odd or even. In this
case, highest derivative that gives zero value at x0 has derivative power n.
There is a saddle point at x0 if n is even.
Bounding Values If there are extrema points within closed range [a, b],
then the extreme values (positive or negative) of the function are also known
10 Maxima  Minima
as bounding values of the function. For example, suppose function
f(x) = x − x2
which is continuous and differentiable at every point defined within (0, 1).
The function has a extrema at
f
′
(x) = 1 − 2x = 0 ⇒ x = 0.5
Now the function value at x = 0.5 is
f(0.5) = 0.25
Again second derivative
f
′′
(0.5) = −2
is negative value. Hence the extrema is also local maxima. Now the roots
of function are
x − x2
= 0 ⇒ x = 0, 1
Function value at these two points are
f(0) = 0 f(1) = 0
Hence the function value ranges between (0, 0.25) ie in other words the
function is bounded between 0 to 0.25 (upper bound by 0.25 and lower
bound by 0).
1.2.8 Saddle Point
A point on a function f(x) which is a stationary point but not an extremum.
To clarify the point assume an function f(x) = 3x3
. Saddle point is mostly
visible in one-dimensional functions with one term. First derivative of the
function is f
′
(x) = 9x2
. Second derivative of the function is f
′′
(x) = 18x.
And third derivative is f
′′′
(x) = 18. The function has a saddle point at
x0 = 0 by extremum test as1
f
′′
(x0) = 0; f
′′′
(x0) = 18
1
3rd
derivative has non zero value at point x0 while 2nd
derivative has zero value at
x0. 2 power in second derivative is an even number hence there is a saddle point at x0.
1.3. EXTREMUM TEST 11
Where, third derivative is not zero at point x0.
1.3 Extremum Test
To find the extrema of a function f(x) at all points x0, first derivative of func-
tion f(x) is obtained. As the slope must be zero at stationary points hence
the turning points can be calculated by equating first derivative to zero, ie
f
′
(x) = 0. This gives all the values of points x0. To classify a stationary
point extremum test or second derivative test is carried out. If second deriva-
tive of the function f(x) at extremum points is positive (f
′′
(x0)  0) then
gradient is increasing and stationary point is minimum. If second derivative
of the function f(x) is negative (f
′′
(x0)  0) then gradient is decreasing and
stationary point is maximum. If second derivative is zero at stationary point
(f
′′
(x0) = 0) then it is a point of inflection or it could be an extremum. To
find the extrema we differentiate the first derivative until we would not find
the non zero derivative at that stationary point.
1.4 Method of Finding Extremum
There are two methods of finding extremum. (i) First derivative test and (ii)
second derivative test.
1.4.1 First Derivative Test
Suppose a function f(x) that is continuous at a stationary point x0. First
derivative of the function f(x) represents the slope of tangent on the function
at the point ‘x’. For maxima or minima at any point on the function graph,
following rules are observed.
1. If f
′
(xl)  0 on an open interval extending left from x0 and f
′
(xr)  0
on an open interval extending right from x0, then f(x) has a local
maximum at x0. It is also possibility that there is an global maximum.
12 Maxima  Minima
x
y
b
b
b
xl x0 xr
f
′
(xl)  0
f
′
(x0) = 0
f
′
(xr)  0
+θ
−θ
2. If f
′
(xl)  0 on an open interval extending left from x0 and f
′
(xr)  0
on an open interval extending right from x0, then f(x) has a local
minimum at x0. It is also possibility that there is an global minimum.
x
y
b
b
b
xl x0 xr
f
′
(xl)  0
f
′
(x0) = 0
f
′
(xr)  0
−θ
+θ
3. If f
′
(x) has the same sign on an open interval extending left from x0
and on an open interval extending right from x0, then f(x) has a point
of inflection at x0.
1.4.2 Second Derivative Test
Suppose a function f(x) is continuous and twice differentiable at stationary
point x0. There are two cases of the f
′′
(x) at point x0.
1. If f
′′
(x0)  0, then slope of tangent on function f
′
(x0) is upward and
the function f(x0) has a relative minima at stationary point x0.
2. If f
′′
(x0)  0, then slope of tangent on function f
′
(x0) is downward
and the function f(x0) has a relative maxima at stationary point x0.
1.4. METHOD OF FINDING EXTREMUM 13
x
y
b
b
b
xl x0 xr
f
′
(xl)
f
′
(x0)
f
′
(xr)
(a)
x
y
b
b
b
xl x0 xr
f
′′
(xl) f
′′
(x0)
f
′′
(xr)
(b)
−θ
(c)
Figure 1.3: Figure (a) graphed presentation of first derivative of sin(x), (b)
graphed presentation of second derivative of sin(x) and (c) slope of second
derivative of sin(x) at point x0.
1.4.3 Why Second Derivative Required
Take a function f(x) whose curve is shown in the following figure. It has
four inversion points at x = −2, x = −1, x = 0, x = 1 and x = 2. First
derivative of function f(x) is plotted too. f
′
(x) is plot of numeric values of
the tangents on functions f(x). It means, f
′
(x) should have zero numeric
value where function f(x) has function inversion. This is why, to get the
inversion points of function f(x), first derivative of function should be equal
to zero. It gives all inversion points but it does not tell about the point that
whether it is maxima or it is minima. The first derivative of function, i.e.
f
′
(x) clearly tells about the nature of function. When f
′
(x)  0, function is
increasing and when f
′
(x)  0 function is decreasing.
x
f(x)
f
f
′
x
=
1.58
x
=
0
x
=
−1.58
(a)
x
f(x)
f
f
′
f
′′
x
=
1.58
x
=
0
x
=
−1.58
(b)
When function changes from decreasing to increasing, the numeric value
of f
′
(x) changes from negative value to positive value. It means, f
′
(x) has
zero crossing at point of inversion and function plot has slop too. If this slope
is positive then f
′′
(x) should be positive value. Similarly, when function
changes from increasing to decreasing, the numeric value of f
′
(x) changes
14 Maxima  Minima
from positive value to negative value. It means, f
′
(x) has zero crossing at
point of inversion and function plot has slop too. If this slope is negative then
f
′′
(x) should be negative value. Hence, where f
′′
(x) is negative, then at that
point function has maxima and where f
′′
(x) is positive, then at that point
function has minima. This is the reason, that second derivative is required
to verify the maxima and minima point.
Solved Problem 1.3 A company produces auto parts and earns income from
sale of these parts. The earning from sale of auto parts is based on the labour
(l) employed and capital (c) invested by the company. Earning function is
I = 2l + 4c − 3l2
− 2c2
+ 9lc
Find the quantity of labour employed and amount of capital invested by the
company to earn the maximum income.
Solution The variation in the income of the company when there is
variation in labour employed or capital invested is maximum or minimum
when dI/dl = 0 and dI/dc = 0. So,
dI
dl
=
d
dl
2l + 4c − 3l2
− 2c2
+ 9lc

= 2 − 6l + 9c = 0
And
dI
dc
=
d
dc
2l + 4c − 3l2
− 2c2
+ 9lc

= 4 − 4c + 9l = 0
1
0.5 1.0 1.5 2.0 2.5 3.0 3.5 4.0 4.5 5.0
l c
I
l c
Two relations of labour and capital are
6l − 9c = 2; 9l − 4c = 4
On solving these equations, we have l = 68/171 and c = 2/19. Again, we
check whether income is maximum at these values of labour employed and
1.5. EXTREMUM IN TWO-DIMENSIONAL FUNCTION 15
capital invested. So, we shall find d2
I/dl2
and d2
I/dc2
. So,
d2
I
dl2
= −6;
d2
I
dc2
= −4
As both are negative, hence, income shall be maximum at this labour em-
ployed and capital invested.
1.5 Extremum in Two-Dimensional Function
If f(x, y) is a two-dimensional function that has a relative extremum at a
point (x0, y0) and has continuous partial derivatives at this point, then
∂f(x0, y0)
∂x
= 0
and
∂f(x0, y0)
∂y
= 0
The second partial derivatives test classifies the point as a local maximum
or relative minimum. The second derivatives of the function is D then
D = fxxfyy − f2
xy
Now
1. If D  0 and fxx(x0, y0)  0, the point is a relative minimum.
2. If D  0 and fxx(x0, y0)  0, the point is a relative maximum.
3. If D  0, the point is saddle point.
4. If D = 0, higher order tests must be used.
Note that, if we have not given the points x0 and y0 then we can find these
points where extremum may exist by solving the equations obtained by using
relation ∂f/∂x = 0 and ∂f/∂y = 0.
16 Maxima  Minima
1.6 Applied Problems
Solved Problem 1.4 Find the maximum value of y = x3
− 9x.
Solution The given function y is zero when x = 0. To get the critical
points where tangent on the function is parallel to abscissa, we derivative the
function with respect to variable ‘x’. Now
dy
dx
= 3x2
− 9
0 = 3x2
− 9
On solving, the critical points are x = ±
√
3.
x
f(x) f
(
x
)
=
x
3
−
9
x
b
f
′
(x) = 0
b
f
′
(x) = 0
(a)
x
f(x)
f
(
x
)
=
x
3
−
9
x
b
f(−
√
3)
b
(−
√
3, 0)
b
f(
√
3)
b
(
√
3, 0)
(b)
Figure 1.4:
From figure 1.4(b), function is continuously decreasing when x ≤ −
√
3
and continuously increasing when x ≥
√
3. In the domain of −
√
3 ≤ x ≤
√
3
function is decreasing. To verify that these points are at minima or maxima
location, second derivative value of the function y is obtained at these critical
points.
d2
y
dx2
= 6x
At x = −
√
3, second derivative is negative while at x =
√
3, second derivative
is positive. Now the maxima exists at x = −
√
3 and maximum function value
is
y−
√
3 = −3
√
3 + 9
√
3 = 6
√
3
This is the local maximum value of the function. But from the figure 1.4(b)
it is clear that the minimum value of the function is −∞ and maximum value
of the function is +∞ globally.
1.6. APPLIED PROBLEMS 17
Solved Problem 1.5 Find the maximum value of f(x) = 6x − x2
.
Solution The given function is f(x) = 6x − x2
. To find the maximum
value of the function, we must knew the point where function is maximum.
For this, f
′
(x) = 0.
6 − 2x = 0 ⇒ x = 3
There is only one extrema point.
x
f(x)
f
(
x
)
=
6
x
−
x
2
b
f
′
(x) = 0
(a)
x
f(x)
f
(
x
)
=
6
x
−
x
2
b
f(3)
b
(3, 0)
(b)
At x = 3 function shall be either minima or maxima. To check the
extremum of the function, we shall find f
′′
(x)
x=3
. Now,
f
′′
(3) = −2  0
Hence function is maxima at x = 3. Now the function value at x = 3 is
f(3) = 6 × 3 − 32
= 9
This is result. Note that, function has only one critical point where extrema
function is maximum. Hence there is no definite minimum function value.
Solved Problem 1.6 Find the minimum value of f(x) = x2
− 4x + 4.
Solution The given function is f(x) = x2
−4x+ 4. To find the minimum
value of the function, we must knew the point where function is minimum.
For this, f
′
(x) = 0.
2x − 4 = 0 ⇒ x = 2
There is only one extrema point.
18 Maxima  Minima
x
f(x)
b
f
′
(x) = 0
(a)
x
f(x)
b
f(2)
b
(2, 0)
(b)
At x = 2 function shall be either minima or maxima. To check the
extremum of the function, we shall find f
′′
(x)
x=2
. Now,
f
′′
(2) = 2  0
Hence function is minima at x = 2. Now the function value at x = 2 is
f(2) = 22
− 4 × 2 + 4 = 0
This is result. Note that, function has only one critical point where extrema
function is minimum. Hence there is no definite maximum function value.
Solved Problem 1.7 Find the minimum value of f(x) = x
4
− x3
5
.
Solution The given function is
f(x) =
x
4
−
x3
5
This function has extremum where f;
(x) = 0. So,
1
4
−
3x2
5
= 0 ⇒ 5 − 12x2
= 0
It gives x = −0.65 and x = +0.65. This function has two extrema. Hence
we shall test whether function minima or function maxima exits.
x
f(x)
b
f
′
(x) = 0
b
f
′
(x) = 0
(a)
x
f(x)
b
f(0.65)
b
(0.65, 0)
b
f(−0.65)
b
(−0.65, 0)
(b)
1.6. APPLIED PROBLEMS 19
To test the extrema, we shall find the f
′′
(x) at x = ±0.65. So,
f
′′
(x) = −
6x
5
f
′′
(x) is negative at x = 0.65, hence function maxima is at x = 0.65. f
′′
(x)
is positive at x = −0.65. hence function is minima at x = −0.65. The
minimum function value is
f(−0.65) =
−0.65
4
−
(−0.65)3
5
= 0.107575
and the function maximum value is
f(0.65) =
0.65
4
−
(0.65)3
5
= 0.107575
Hence, function minimum value is −0.10 approximately.
Solved Problem 1.8 Find the maximum value of f(x) = 2
√
x − 2x + 3.
Solution To get the critical points of the function, we should put f
′
(x) =
0. So,
2 ×
1
2
×
1
√
x
− 2 = 0
This give √
x = 0.5 ⇒ x = 0.25
x
f(x)
b
f
′
(x) = 0
f
=
2 √
x
−
2x
+
3
(a)
x
f(x)
b
f(1/4)
b
(1/4, 0)
f
=
2 √
x
−
2x
+
3
(b)
This has only one x = 0.25, where one extrema exists. To check whether
function is maximum or minimum at this point, we shall find f
′′
(0.25). So,
f
′′
(x) = −
1
2
×
1
3
√
x
 0; ∀x ∈ R+
20 Maxima  Minima
At x = 0.25, f
′′
(0.25) is negative, hence function is maxima at x = 0.25. The
function value at this point is given by
f(0.25) = 2 ×
√
0.25 − 2 × 0.25 + 3 = 3.5
This is desired result.
Solved Problem 1.9 Find the maximum value of f(x) = 6x − x2
.
Solution To get the critical points of the function, we should put f
′
(x) =
0. So,
6 − 2x = 0 ⇒ x = 3
x
f(x)
f(x) = 6x − x2
b
f
′
(x) = 0
(a)
x
f(x)
f(x) = 6x − x2
b
f(3)
b
(3, 0)
(b)
This has only one x = 3, where one extrema exists. To check whether
function is maximum or minimum at this point, we shall find f
′′
(3). So,
f
′′
(x) = −2  0; ∀x
At x = 3, f
′′
(3) is negative, hence function is maxima at x = 3. The function
value at this point is given by
f(3) = 6 × 3 − 32
= 9
This is desired result.
Solved Problem 1.10 Find the minimum value of f(x) = cos(3x) in range of
[0, π/2].
Solution The given function is f(x) = cos(3x). The tangents on the
function at its extrema will be there where f
′
(x) = 0. So,
−3 sin(3x) = 0 ⇒ sin(3x) = 0
1.6. APPLIED PROBLEMS 21
Note that, function is a periodic function, hence it shall have infinite numbers
of extrema and corresponding critical points. But we are taking here only
local extrema, i.e. maximum and minimum values within definite domain.
x
f(x)
f(x) = cos(3x)
b
b
f
′
(x) = 0
f
′
(x) = 0
(a)
x
f(x)
f(x) = cos(3x)
b
b
f (0)
f (1.046)
b b
(0, 0)
(1.046, 0)
(b)
Now, 3x = nπ or x = nπ/3. We have to find the minimum value in
x ∈ [0, π/2]. For this domain, only accepted critical points2
are found when
n = 0 and n = 1. So, x is 0 and π/3. To check where, function is minimum,
we have to find f
′′
(x)  0. Now
f
′′
(x) = −9 cos(3x)
f
′′
(x) is positive when cos(3x) is negative, i.e. π/2 ≤ 3x ≤ π or π/6 ≤ x ≤
π/3. Note that we are talking about cos(3x) of f
′′
(x) not about the cos(3x)
of f(x). So, only acceptable value for minima from x = 0 and x = π/3 is
x = π/3. Now minimum function value is
f(π/3) = cos

3 ×
π
3

= −1
This is desired result.
Solved Problem 1.11 Find the maximum value of y = sin(x) + tan(x) in
range of (−2π, 2π).
Solution The given function is y = sin(x) + tan(x). To get the critical
points, we should put y
′
(x) = 0. Now,
cos(x) + sec2
(x) = 0
On solving it, we have
cos3
(x) = −1
2
Roots are those points where function is zero. Critical points are those points where
tangent on the function is zero.
22 Maxima  Minima
This has three roots, in which only one is real. The real root is cos(x) = −1.
On solving it, we have
cos(x) = cos(2n + 1)π ⇒ x = (2n + 1)π
Here, n is integer. The values of x withing (−2π, 2π) is −π and π only when
n = −1 and n = 0. These are only points where extrema exist.
x
y(x)
y(x) = sin(x) + tan(x)
b b
y
′
(x) = 0 y
′
(x) = 0
(a)
x
y(x)
y(x) = sin(x) + tan(x)
f (−π) f (π)
b b
b b
(−π, 0) (π, 0)
(b)
To check the nature of extrema, we shall get the value of y
′′
(x) at x = −π
and x = π. Hence
y
′′
(x) = 2 sec2
(x) × tan(x) − sin(x)
At x = −π
y
′′
(−π) = 2 sec2
(−π) × tan(−π) − sin(−π) = 0
At x = π
y
′′
(π) = 2 sec2
(π) × tan(π) − sin(π) = 0
At both critical points, y
′′
(x) is zero. Hence function has reflection at points
x = ±π.
Solved Problem 1.12 Find the range of θ for sine function of unit amplitude
so that the bounded rectangular area is maximum.
Solution
θ
y
b
θ
b
f(θ)
b
π − θ
b
f(θ)
θ
y
1.6. APPLIED PROBLEMS 23
As per the symmetry of the sine function of unit amplitude, shape of
maximum area will be in rectangular shape as shown in above figure. Area
of this shape is
A = f(θ) × (π − 2θ)
On derivating it with respect to θ and equating it to zero, it gives
tan(θ) + θ −
π
2
= 0
The solution of this relation gives the critical value of θ within the limit of
0  θ  π is θ = 0.7104. Second derivative of the area is
A′′
= − [cos θ − (π − 2θ) sin θ − 2 cos θ]
It is negative when θ is substituted. Area will be maximum at the criti-
cal value of θ = 0.7104. Maximum area is bounded between the limits of
0.7104 ≤ θ ≤ 2.4311.
Solved Problem 1.13 Find the minimum and maximum value of f(x) =
sin2
(x) in range of [0, π/2].
Solution The critical points of the function are found by putting f
′
(x) =
0. So,
2 sin(x) × cos(x) = 0 ⇒ sin(2x) = 0
This gives
sin(2x) = sin(nπ) ⇒ x = n
π
2
The values of x lies in [0, π/2] are x = 0 and x = π/2.
x
f(x)
f(x) = sin2
(x)
b
b
f
′
(x) = 0
f
′
(x) = 0
(a)
x
f(x)
f(x) = sin2
(x)
b
b
f π
2

f (0)
b
b
π
2
, 0

(0, 0)
(b)
The extrema of function at these critical points are either maxima or
minima. To check this, we have to find sign of f
′′
(x).
f
′′
(x) = 2 cos(2x)
24 Maxima  Minima
At x = 0, f
′′
(0) = 2  0, hence function has minima at x = 0. Similarly at
x = π/2, f
′′
(π/2) = −2  0, hence function has maxima at x = π/2. The
minima and maxima values are respectively
f(0) = [sin(0)]2
= 0
and
f(π/2) = [sin(π/2)]2
= 1
This is desired result.
Solved Problem 1.14 Find the minimum and maximum value of f(x) =
sin(x2
) in range of [0, π/2].
Solution To get the extrema of the function, we should put f
′
(x) = 0.
Now, it gives
cos(x2
) × 2x = 0
This gives x = 0 and cos(x2
) = cos((2n + 1)π/2). Or
x2
=
(2n + 1)π
2
This gives,
x = ±
r
(2n + 1)π
2
x
f(x)
f(x) = sin(x2
)
b
b
b
f
′
(x) = 0
f
′
(x) = 0
f
′
(x) = 0
(a)
x
f(x)
f(x) = sin(x2
)
b
b
b
f

−
q
π
2

f (0)
f
q
π
2

b b b

−
q
π
2
, 0

(0, 0)
q
π
2
, 0

(b)
Values of x in the domain [0, π/2] are x = 0 and at n = 0. x = +
p
π/2.
x = −
p
π/2 is not acceptable as it is out of domain. To check where function
is maximum and where function is minimum, we have to check sign of f
′′
(x).
So
f
′′
(x) = cos(x2
) × 2 + 2x × − sin(x2
) × 2x
Or
f
′′
(x) = 2 cos(x2
) − 4x2
sin(x2
)
1.6. APPLIED PROBLEMS 25
This value at x = 0 and x =
p
π/2 are
f
′′
(0) = 2 cos(02
) − 4 × 02
× sin(02
) = 2  0
f
′′
(
p
π/2) = 2 cos(π/2) − 4 ×
π
2
× sin(π/2) = −2π  0
Hence function is maximum at x =
p
π/2 and minimum at x = 0. So
maximum and minimum function values are respectively:
f
r
π
2

= sin
π
2

= 1
f(0) = sin(0) = 0
Solved Problem 1.15 Is it possible to find the maximum or minimum values
of the function f(x) = tan(2x) in range of [0, π/2]?
Solution If tangents on the curve are not parallel to horizontal axis in
the given domain [0, π/2], then function is not changing its direction from
increasing to decreasing or vice-versa in the given domain [0, π/2]. If there
is a point within the domain [0, π/2] at which tangent is parallel to x-axis,
then we must put f
′
(x) = 0 and find the critical points. If there are nay
any critical point within the domain [0, π/2], then there is no tangent that
is parallel to horizontal axis and exists within the domain [0, π/2].
x
f(x)
f(x) = tan(2x)
b b
(0, 0)
π
2
, 0

So,
f
′
(x) = sec2
(2x) × 2 = 0
This gives
sec2
(2x) = 0 ⇒ 1 + tan2
(2x) = 0
26 Maxima  Minima
Or
tan2
(2x) = −1 ⇒ tan(2x) =
√
−1 = i
Now, there are no points of x within given domain for which tangent satisfy
above condition. Hence tangents on the given function are not parallel to
the x-axis. Hence it is not possible to find the maximum or minimum value
of the given function at given domain.
Solved Problem 1.16 Is function f(x) = tan(2x) is strictly increasing in
range of [0, π/2]?
Solution For any two points a, b ∈ I, if a function f(x) satisfy the
relation f(b)  f(a), then it is said to be strictly increasing function for all
b  a. If function has f(b) ≥ f(a) for any two values of a and b then we
said that function is nonstrictly increasing. So, for the given function, we
first check whether there is minima or maxima of the function in [0, π/2]. If
there is so, two points either side of the extrema shall give equal values of
function. So,
f
′
(x) = 2 sec2
(2x) = 0
The range of secant is [1, ∞). So, for any value of x, there is never zero value
of secant. So, no point exits in [0, π/2] at which there is extrema on the given
function f(x). Now, for two extreme values, x = 0 and x = π/2, function
values are
f(0) = tan(0) = 0; f(π/2) = tan(2 × π/2) = tan(π) = 0
Here, two values f(0) and f(π/2) are equal, hence function is not strictly
increasing in the given domain x ∈ [0, π/2].
Solved Problem 1.17 Income of a company is represented by I = x3
−cx+3.
Find the maximum income of the company when c = 3.
Solution The income of a company is given by
I = x3
− cx + 3
This income curve shall be maximum or minimum, where I
′
= 0. So,
3x2
− c = 0 ⇒ ±x =
r
c
3
1.6. APPLIED PROBLEMS 27
The income shall be maximum at x if I
′′
(x)  0. So,
I
′′
(x) = 6x
This is negative at x = −
pc
3
, hence income is maximum at x = −
pc
3
.
Substituting c = 3, we have, x = −1 Now, maximum income of the company
is
Imax = (−1)3
− 3 × −1 + 3 = 5
This is maximum income of the company.
Solved Problem 1.18 Marks of a student is represented by M = 9x2
−3x+5.
Find the maximum marks of the student.
Solution The maximum marks is given by
M = 9x2
− 3x + 5
This mark curve shall be maximum or minimum, where M
′
= 0. So,
18x − 3 = 0 ⇒ x =
1
6
The marks shall be maximum at x if M
′′
(x)  0. So,
M
′′
(x) = 18  0
This is positive at x = 1/6 and it is minima condition. Hence there is no
upper limit of maximum marks. So, this marks curve tells that student has
no definite maximum marks.
Solved Problem 1.19 Sum of two arbitrary numbers is 24. Find the minimum
value of product of these two arbitrary numbers if product is given by p =
xy2
.
Solution The two numbers are x and y. According the question, x+ y =
24. The product function is
p = xy2
= (24 − y)y2
This product curve shall be maximum or minimum, where p
′
= 0. So,
48y − 3y2
= 0 ⇒ 3y(16 − y) = 0
28 Maxima  Minima
This gives, y = 0 or y = 16. y = 0 is not acceptable as it make product zero.
Other number is y = 16. We shall check whether product is maximum for
y = 16. For this p
′′
(y)  0. So,
p
′′
(y) = 48 − 6y
This is negative, at y = 16, hence product shall be maximum at y = 16. Now
the other number is x = 24 − 16 = 8. The product is
p = 8 × 162
= 2048
This is desired result.
Solved Problem 1.20 Velocity of a moving particle in horizontal plane is
represented by v = x3
− 4x. Here x is an arbitrary parameter. Find the
maximum velocity of the particle.
Solution The velocity function is
v = x3
− 4x
This velocity curve shall be maximum or minimum, where v
′
= 0. So,
3x2
− 4 = 0 ⇒ ±x =
r
4
3
We shall check where velocity is maximum. For this v
′′
(x)  0. So,
v
′′
(x) = 6x
v
′′
(x) is negative at x = −
q
4
3
, hence velocity shall be maximum at this
point. Now, velocity is
v = −
r
4
3
!3
− 4 × −
r
4
3
=
16
3
√
3
This is desired result.
1.6. APPLIED PROBLEMS 29
1.6.1 Hypothetical Extrema
There are some hypothetical extrema exist in a function. For example, a
time function h = t3
− 6t − 8 has an exremum point at t = −
√
2. But in
timeline, this value never exits as time always started at t = 0. See the
following example:
Solved Problem 1.21 A container is filled by water dripping from a tap. The
height of water in container is a time function and it is given by h = t3
−6t+8.
Find the maximum rate of water filling to the container.
Solution The water filling function is
h = t3
− 6t − 8 = f(t)
This function gives f(−∞) = −∞ and f(∞) = ∞. Hence this function have
global and may have local extrema. Function values at local extrema tell us
whether water filling is maximum or minimum in definite time duration. So,
to find the critical points of extrema, we have f
′
(t) = 0. So,
3t2
− 6 = 0
It gives two critical points, t =
√
2 or t = −
√
2.
t
f(t)
f
(
t
)
=
t
3
−
6
t
−
8
b
b
f
′
(t) = 0
f
′
(t) = 0
(a)
t
f(t)
f
(
t
)
=
t
3
−
6
t
−
8
b
b
f(
√
2)
f(−
√
2)
b
b
√
2, 0

−
√
2, 0

(b)
To check whether function is maxima or minima at a given critical point,
f
′′
(t)  0 or f
′′
(t)  0 respectively. Now
f
′′
(t) = 6t
This is negative at t = −
√
2 and at this point filling function shall be maxi-
mum. So filling is maximum at time t = −
√
2. This time never exists in time
line, hence maximum water filling rate shall never achieved at local extrema.
30 Maxima  Minima
Solved Problem 1.22 A moving distance relation for a particle is y = t3
−
6 ∗ t2
+ 11 ∗ t − 6. Find the extremum speed of the particle if possible.
Solution The distance relation is y = t3
−6t2
+ 11t−6. Velocity relation
will be derivative of the distance relation y. So,
v =
d
dt
y = 3t2
− 12t + 11
This velocity curve has extrema points. To find these extrema points, we
will get v
′
(t) = 0. So
v
′
(t) = 6t − 12 = 0
This gives t = 2, There is only one extrema point, hence there will be either
minima or maxima at this point.
x
y(x) v(x)
y(x)
b
v
′
(x) = 0
(a)
x
y(x) v(x)
y(x)
b
v(2)
b
(2, 0)
(b)
To check, there is maxima or minima, get v
′′
(t).
v
′′
(t) = 6  0
This is positive value irrespective of t. So, there shall be minimum velocity
at t = 2. The minimum velocity is
v(2) = 3 × 22
− 12 × 2 + 11 = −1
This is desire result.
Solved Problem 1.23 Find the dimensions of rectangle inscribed in semi-
circle of radius r having maximum area.
Solution Let there is semi-circle of radius r. A rectangle is inscribed in-
side this semi-circle as shown in first part of below figure. Let the dimensions
of the rectangle are 2x and y.
1.6. APPLIED PROBLEMS 31
1
1
−1
−2
x
y
b
r
x
y
2
−2
1
−1
x
A
Now area of the rectangle is
A = 2xy
This is two variable relation. We can convert it to one variable relation if x
and y are given in relation with r (constant). From the first part of above
figure
r2
= x2
+ y2
Replacing y in the area relation, we have
A = 2x
√
r2 − x2
The area is maximum, where x gives maxima point on the area curve. To
get critical points, put A
′
(x) = 0.
A
′
(x) = 2x ×
1
2
× (r2
− x2
)−1/2
× −2x + 2
√
r2 − x2 = 0
On solving it, we have
x2
=
r2
2
⇒ ±x =
r
√
2
⇒ x = ∓
r
√
2
1
1
−1
−2
x
y
b
r
x
y
1
1
−1
−2
x
y
√
2 r
r/
√
2
To check whether area is maximum or minimum at x = ∓ r
√
2
, we have to
find the sign of A
′′
(x) at the given x points.
A
′′
(x) = −
6x
√
r2 − x2
−
2x3
(r2 − x2)
3
2
32 Maxima  Minima
Value of A
′′
(x) at x = ∓ r
√
2
are
A
′′

r
√
2

= −
6 ×

r
√
2

r
r2 −

r
√
2
2
−
2 ×

r
√
2
3

r2 −

r
√
2
2
3
2
 0
At this point area is maximum. Similarly,
A
′′

−
r
√
2

= −
6 ×

− r
√
2

r
r2 −

− r
√
2
2
−
2 ×

− r
√
2
3

r2 −

− r
√
2
2
3
2
 0
At this point area is minimum. Now, the maximum area of the inscribe
rectangle is at x = r
√
2
. Now, the maximum area of inscribed rectangle is
A = 2 ×
r
√
2
×
s
r2 −

r
√
2
2
= r2
The dimensions of the rectangle are 2x =
√
2 r and y =
√
r2 − x2 = r/
√
2.
Solved Problem 1.24 Cost of production of x articles is given by c(x) = 2x3
−
4x2
+4 and revenue received from the selling of same articles is r(x) = x2
−1.
Find the production of articles when profit is maximum.
Solution
1.6. APPLIED PROBLEMS 33
x
p(x)
c(x) = 2x3
− 4x2
+ 4
r(x) = x2
− 1
b
b
b
c
′
(x) = 0
c
′
(x) = 0
r
′
(x) = 0
(a)
x
p(x)
c(x) = 2x3
− 4x2
+ 4
r(x) = x2
− 1
b
b
b
c(0)
c(1.33)
r(0)
b b
b
(0, 0)
(1.33, 0)
(b)
Figure 1.5:
If cost of production of ‘x’ items is c(x) and revenue received from selling
of ‘x’ articles is r(x) then profit of selling of ‘x’ items is p(x) = r(x) − c(x).
If r(x), c(x) are differentiable for all x  0 then maximum profit occurs at
p
′
(x) = 0. Now
d p(x)
dx
=
d r(x)
dx
−
d c(x)
dx
At maximum profit
d p(x)
dx
= 0
ie
d r(x)
dx
=
d c(x)
dx
Substituting the values of r(x) and c(x), we have
2x = 6x2
− 8x
On solving it
x = 0 or
=
5
3
≈ 1.666
The second derivatives of r(x) and c(x) at this value of ‘x’ are
d2
r(x)
dx2
= 2 = +ve
34 Maxima  Minima
and
d2
c(x)
dx2
= 12x − 8 = +ve
respectively. The profit is maximum when production units are near about
1.666 in factor.
Solved Problem 1.25 A cabinetmaker needs wood to produce 5 furnishings
each day. He orders wood from woodseller periodically. The delivery cost
of wood is Rs. 5000 per supply order. The storage cost of one furnished
cabinet is Rs. 10 per day. Considering the delivery cost and storage charges,
calculate the period after which he should order to minimize his average daily
cost. Also calculate the quantity of wood to be delivered in one delivery.
Solution
x
c(x)
y = 5000
x
y = 25 · x
c(x) = 5000
x
+ 25 · x
x-min
Figure 1.6:
Assume the cabinetmaker should order wood after ‘x’ days. In ‘x’ days
he will produce total furnishings ‘5x’. The average cost of storage S(x) of
furnished cabinet for ‘x’ days is product of furnished cabinets, number of
days to be stored, cost of storage per unit per day and divided by two (for
average).
S(x) =

5x
2

· x · 10
Total cost T(x) of delivery and storage during two consecutive delivery is
T(x) = 5000 +

5x
2

· x · 10
1.6. APPLIED PROBLEMS 35
Average daily cost C(x) during the two consecutive delivery is
C(x) =
5000
x
+ 25x, x  0
When x → 0 or x → ∞, the average daily cost becomes large, hence the
minimum value of ‘x’ should exists in between these two limits ie 0  x  ∞.
Now to get the critical point, differentiate cost relation with respect to ‘x’.
d C(x)
dx
= −
5000
x2
+ 25
On solving, two critical points are x = ±
√
200 ≈ ±14. Only x =
√
200 ≈ 14
lies in the domain of ‘x’. To verify the minimum cost, second derivative of
average day cost at x = 14 is

d2
C(x)
dx2

14
=

10000
x3

14
= +ve
This means, cabinet maker should ask for delivery of wood in period of 14
days. The average daily cost in rupees is
C(
√
200) =
5000
√
200
+ 25 ×
√
200 ≈ 707
The quantity of wood should schedule a delivery is Q = 5 · 14 = 70 units.
Solved Problem 1.26 Prove that the maximum volume of a cone that is
inscribed in a sphere of radius R is 8
27
times of the volume of sphere.
Solution Let a right circular cone is constructed inside a sphere whose
radius is R. Base radius of right circular cone is r and height is h. Semi-
vertical angle of cone is θ (say).
36 Maxima  Minima
r
h
l
l
2
R
R
b
O
A
B C
D
E
θ
Figure 1.7: Inscribing a right circular cone in a sphere of radius R.
Now volume of cone is given by
V =
1
3
πr2
h (1.1)
Both r and h are variables. Converting radius and height of cone if form of
radius of sphere. From figure
sin θ =
l
2
R
That gives
l = 2R sin θ (1.2)
Again
sin θ =
r
l
Here l is slant height of the right circular cone.
cos θ =
h
l
1.6. APPLIED PROBLEMS 37
Substituting the values of r and l in equation (1.1)
V =
1
3
πl2
sin2
θ l cos θ (1.3)
Substituting the value of l in above relation from equation (1.2), volume of
cone is
V =
8
3
πR3
cos4
θ sin2
θ (1.4)
To find the maximum volume differentiate to the equation (1.4) with respect
to θ and equation it to zero.
d
dθ
V =
d
dθ

π
8
3
R3
cos4
θ sin2
θ

Applying product rule for differentiation in right hand side of above equation.
Hence
d
dθ
V = π
8
3
R3

cos4
θ 2 sin θ cos θ − 4 cos3
θ sin θ sin2
θ

= 0
On simplification
2 cos2
θ = 4 sin2
θ
This gives
tan θ =
1
√
2
(1.5)
Applying right angle triangle rule
sin θ =
1
√
3
And
cos θ =
√
2
√
3
Now for maximum volume second derivative test for volume of cone must be
negative for the value of sin θ and cos θ. Now second derivative value of cone
volume is
d2
dθ2
V =
d2
dθ2

π
8
3
R3
cos4
θ sin2
θ
38 Maxima  Minima
Second derivative is
d2
dθ2
V =
8
3
πR3

12 cos2
θ sin4
θ − 22 cos4
θ sin2
θ + 2 cos6
θ

Substituting the values of cos θ and sin θ in above relation
d2
dθ2
V =
8
3
πR3

24
27
−
88
27
+
16
27

= −ve (1.6)
This means volume of cone is maximum for given values of cos θ and sin θ.
Now Substituting the values of cos θ and sin θ in relation (1.1)
V =
8
3
πR3 4
9
1
3
V =
8
27
πR3 4
3
| {z }
Volume of Sphere
(1.7)
This gives that maximum volume of cone is 8
27
times of the volume of sphere.
Solved Problem 1.27 Find the maximum area of rectangle that is inscribed
in ellipse x2
a2 + y2
a2 = 1.
Solution A rectangle is drawn inside the ellipse as its four corners are at
the ellipse. Assume a point A whose coordinate is (x, y). From symmetry
other three coordinates of corners of rectangle are B(x, −y), C(−x, −y) and
D(−x, y). Now length of side AB of rectangle is 2y and length of side BC
is 2x. Area of rectangle is
1.6. APPLIED PROBLEMS 39
x
y
O
A
B
C
D
(x, y)
(x, −y)
(−x, −y)
(−x, y) b
b
b
b
2y
2x
Figure 1.8: Inscribing a rectangle in a ellipse.
A = 2x × 2y (1.8)
From the definition of ellipse x2
a2 + y2
a2 = 1
y =
b
a
√
a2 − x2 (1.9)
Now the area of rectangle is
A = 4x
b
a
√
a2 − x2 (1.10)
For maximum or minimum area of rectangle, first derivative of equation
(1.10) with respect to x would be make equal to zero to find the extrema
points. ie
dA
dx
= 4 ·
b
a
·

√
a2 − x2 −
x2
√
a2 − x2

For stationary values of x equating above relation to zero and solving for x.
The result is
a2
− x2
= x2
x = ±
a
√
2
40 Maxima  Minima
If are of rectangle is extremum then second derivative test of area A should
be negative
d2
A
dx2
= 4 ·
b
a
·

−
3 x
√
a2 − x2
−
x3
(a2 − x2)
3
2
#
(1.11)
On substituting the value of x, d2A
dx2 is negative, hence area of the rectangle
inscribed inside the ellipse is maximum when
x = ±
a
√
2
Now maximum area of inscribed rectangle is
A = 4 ·
b
a
·
a
√
2
s
a2 −

a
√
2
2
(1.12)
On solving equation (1.12), maximum area of rectangle is
A = 2ab (1.13)
Solved Problem 1.28 Find the maximum area of isosceles triangle that is
inscribed in ellipse x2
a2 + y2
a2 = 1.
Solution A isosceles triangle is drawn inside the ellipse as its tree vertices
are at the ellipse. Assume a point A whose coordinate is (a, 0). From sym-
metry of isosceles triangle other three coordinates are B(−x, −y), C(−x, y)
and D(−x, 0). Now length of height AD of triangle is (x + a) and length of
base BC of triangle is 2y. Area of triangle is
1.6. APPLIED PROBLEMS 41
x
y
b
b
b
b
O A
B
C
D
(a, 0)
(−x, −y)
(−x, y)
(−x, 0)
a + x
2y
Figure 1.9: Inscribing a isosceles triangle in a ellipse.
A =
1
2
2y × (x + a) (1.14)
From the definition of ellipse x2
a2 + y2
a2 = 1
y =
b
a
√
a2 − x2 (1.15)
Now the area of triangle is
A = (x + a) ×
b
a
√
a2 − x2 (1.16)
To find maximum or minimum area of triangle, first derivative of equation
(1.16) with respect to x would be make equal to zero for extrema points. ie
dA
dx
=
b
a
·

√
a2 − x2 −
x(x + a)
√
a2 − x2

For stationary values of x equating above relation to zero and solving for x.
The result is
2x2
+ ax − a2
= 0
Solving it by using Shridharacharya formula (quadratic solution method)
x =
a
2
, −a

More Related Content

What's hot

3.5 Rational Functions
3.5 Rational Functions3.5 Rational Functions
3.5 Rational Functionssmiller5
 
Density theorems for anisotropic point configurations
Density theorems for anisotropic point configurationsDensity theorems for anisotropic point configurations
Density theorems for anisotropic point configurationsVjekoslavKovac1
 
X2 T05 04 reduction formula (2010)
X2 T05 04 reduction formula (2010)X2 T05 04 reduction formula (2010)
X2 T05 04 reduction formula (2010)Nigel Simmons
 
Conditional Statements
Conditional StatementsConditional Statements
Conditional Statementsmicdsram
 
3.5 extrema and the second derivative
3.5 extrema and the second derivative3.5 extrema and the second derivative
3.5 extrema and the second derivativemath265
 
5.1 anti derivatives
5.1 anti derivatives5.1 anti derivatives
5.1 anti derivativesmath265
 
Sine and Cosine Functions Ppt
Sine and Cosine Functions PptSine and Cosine Functions Ppt
Sine and Cosine Functions PptDevan Tate
 
Limits of a Function
Limits of a FunctionLimits of a Function
Limits of a FunctionJesusDel2
 
Trigonometry: Circular Functions
Trigonometry: Circular FunctionsTrigonometry: Circular Functions
Trigonometry: Circular FunctionsSnowfoot
 
Series solution to ordinary differential equations
Series solution to ordinary differential equations Series solution to ordinary differential equations
Series solution to ordinary differential equations University of Windsor
 
Presentaton on Polynomials....Class 10
Presentaton on Polynomials....Class 10 Presentaton on Polynomials....Class 10
Presentaton on Polynomials....Class 10 Bindu Cm
 
Linear Equations
Linear EquationsLinear Equations
Linear Equationsrfant
 
9.3 Intro to Rational Functions
9.3 Intro to Rational Functions9.3 Intro to Rational Functions
9.3 Intro to Rational FunctionsKristen Fouss
 
basic concepts of Functions
basic concepts of Functionsbasic concepts of Functions
basic concepts of FunctionsTarun Gehlot
 

What's hot (20)

Limit and continuity
Limit and continuityLimit and continuity
Limit and continuity
 
3.5 Rational Functions
3.5 Rational Functions3.5 Rational Functions
3.5 Rational Functions
 
Density theorems for anisotropic point configurations
Density theorems for anisotropic point configurationsDensity theorems for anisotropic point configurations
Density theorems for anisotropic point configurations
 
Numerical Methods
Numerical MethodsNumerical Methods
Numerical Methods
 
X2 T05 04 reduction formula (2010)
X2 T05 04 reduction formula (2010)X2 T05 04 reduction formula (2010)
X2 T05 04 reduction formula (2010)
 
Conditional Statements
Conditional StatementsConditional Statements
Conditional Statements
 
3.5 extrema and the second derivative
3.5 extrema and the second derivative3.5 extrema and the second derivative
3.5 extrema and the second derivative
 
5.1 anti derivatives
5.1 anti derivatives5.1 anti derivatives
5.1 anti derivatives
 
Indeterminate form
Indeterminate formIndeterminate form
Indeterminate form
 
Sine and Cosine Functions Ppt
Sine and Cosine Functions PptSine and Cosine Functions Ppt
Sine and Cosine Functions Ppt
 
Limits of a Function
Limits of a FunctionLimits of a Function
Limits of a Function
 
Trigonometry: Circular Functions
Trigonometry: Circular FunctionsTrigonometry: Circular Functions
Trigonometry: Circular Functions
 
Series solution to ordinary differential equations
Series solution to ordinary differential equations Series solution to ordinary differential equations
Series solution to ordinary differential equations
 
Presentaton on Polynomials....Class 10
Presentaton on Polynomials....Class 10 Presentaton on Polynomials....Class 10
Presentaton on Polynomials....Class 10
 
Linear Equations
Linear EquationsLinear Equations
Linear Equations
 
Transformations
TransformationsTransformations
Transformations
 
9.3 Intro to Rational Functions
9.3 Intro to Rational Functions9.3 Intro to Rational Functions
9.3 Intro to Rational Functions
 
Conic Section
Conic SectionConic Section
Conic Section
 
Operations on function.pptx
Operations on function.pptxOperations on function.pptx
Operations on function.pptx
 
basic concepts of Functions
basic concepts of Functionsbasic concepts of Functions
basic concepts of Functions
 

Similar to Maxima & Minima of Functions - Differential Calculus by Arun Umrao

Limit & continuity
Limit & continuityLimit & continuity
Limit & continuityArun Umrao
 
Limit & Continuity of Functions - Differential Calculus by Arun Umrao
Limit & Continuity of Functions - Differential Calculus by Arun UmraoLimit & Continuity of Functions - Differential Calculus by Arun Umrao
Limit & Continuity of Functions - Differential Calculus by Arun Umraossuserd6b1fd
 
maxima & Minima thoeyr&solved.Module-4pdf
maxima & Minima thoeyr&solved.Module-4pdfmaxima & Minima thoeyr&solved.Module-4pdf
maxima & Minima thoeyr&solved.Module-4pdfRajuSingh806014
 
APPLICATION OF PARTIAL DIFFERENTIATION
APPLICATION OF PARTIAL DIFFERENTIATIONAPPLICATION OF PARTIAL DIFFERENTIATION
APPLICATION OF PARTIAL DIFFERENTIATIONDhrupal Patel
 
0.5.derivatives
0.5.derivatives0.5.derivatives
0.5.derivativesm2699
 
Application of derivatives
Application of derivativesApplication of derivatives
Application of derivativesindu thakur
 
Project in Calcu
Project in CalcuProject in Calcu
Project in Calcupatrickpaz
 
Profit Maximization (stats).ppt
Profit Maximization          (stats).pptProfit Maximization          (stats).ppt
Profit Maximization (stats).pptTrade Expert
 
Introduction to Functions
Introduction to FunctionsIntroduction to Functions
Introduction to FunctionsMelanie Loslo
 
Function Analysis v.1
Function Analysis v.1Function Analysis v.1
Function Analysis v.1Arun Umrao
 
Principle of Function Analysis - by Arun Umrao
Principle of Function Analysis - by Arun UmraoPrinciple of Function Analysis - by Arun Umrao
Principle of Function Analysis - by Arun Umraossuserd6b1fd
 
Introduction to functions
Introduction to functionsIntroduction to functions
Introduction to functionsElkin Guillen
 
267 handout 2_partial_derivatives_v2.60
267 handout 2_partial_derivatives_v2.60267 handout 2_partial_derivatives_v2.60
267 handout 2_partial_derivatives_v2.60Ali Adeel
 
237654933 mathematics-t-form-6
237654933 mathematics-t-form-6237654933 mathematics-t-form-6
237654933 mathematics-t-form-6homeworkping3
 

Similar to Maxima & Minima of Functions - Differential Calculus by Arun Umrao (20)

Limit & continuity
Limit & continuityLimit & continuity
Limit & continuity
 
Limit & Continuity of Functions - Differential Calculus by Arun Umrao
Limit & Continuity of Functions - Differential Calculus by Arun UmraoLimit & Continuity of Functions - Differential Calculus by Arun Umrao
Limit & Continuity of Functions - Differential Calculus by Arun Umrao
 
maxima & Minima thoeyr&solved.Module-4pdf
maxima & Minima thoeyr&solved.Module-4pdfmaxima & Minima thoeyr&solved.Module-4pdf
maxima & Minima thoeyr&solved.Module-4pdf
 
Derivatives
DerivativesDerivatives
Derivatives
 
Jacobians new
Jacobians newJacobians new
Jacobians new
 
APPLICATION OF PARTIAL DIFFERENTIATION
APPLICATION OF PARTIAL DIFFERENTIATIONAPPLICATION OF PARTIAL DIFFERENTIATION
APPLICATION OF PARTIAL DIFFERENTIATION
 
0.5.derivatives
0.5.derivatives0.5.derivatives
0.5.derivatives
 
Application of derivatives
Application of derivativesApplication of derivatives
Application of derivatives
 
Project in Calcu
Project in CalcuProject in Calcu
Project in Calcu
 
Profit Maximization (stats).ppt
Profit Maximization          (stats).pptProfit Maximization          (stats).ppt
Profit Maximization (stats).ppt
 
Limit and continuity
Limit and continuityLimit and continuity
Limit and continuity
 
.
..
.
 
Introduction to Functions
Introduction to FunctionsIntroduction to Functions
Introduction to Functions
 
Function Analysis v.1
Function Analysis v.1Function Analysis v.1
Function Analysis v.1
 
Principle of Function Analysis - by Arun Umrao
Principle of Function Analysis - by Arun UmraoPrinciple of Function Analysis - by Arun Umrao
Principle of Function Analysis - by Arun Umrao
 
Note introductions of functions
Note introductions of functionsNote introductions of functions
Note introductions of functions
 
Introduction to functions
Introduction to functionsIntroduction to functions
Introduction to functions
 
267 handout 2_partial_derivatives_v2.60
267 handout 2_partial_derivatives_v2.60267 handout 2_partial_derivatives_v2.60
267 handout 2_partial_derivatives_v2.60
 
AppsDiff3c.pdf
AppsDiff3c.pdfAppsDiff3c.pdf
AppsDiff3c.pdf
 
237654933 mathematics-t-form-6
237654933 mathematics-t-form-6237654933 mathematics-t-form-6
237654933 mathematics-t-form-6
 

More from ssuserd6b1fd

Notes for c programming for mca, bca, b. tech cse, ece and msc (cs) 1 of 5 by...
Notes for c programming for mca, bca, b. tech cse, ece and msc (cs) 1 of 5 by...Notes for c programming for mca, bca, b. tech cse, ece and msc (cs) 1 of 5 by...
Notes for c programming for mca, bca, b. tech cse, ece and msc (cs) 1 of 5 by...ssuserd6b1fd
 
Decreasing increasing functions by arun umrao
Decreasing increasing functions by arun umraoDecreasing increasing functions by arun umrao
Decreasing increasing functions by arun umraossuserd6b1fd
 
Distribution of normal data understanding it numerical way by arun umrao
Distribution of normal data   understanding it numerical way by arun umraoDistribution of normal data   understanding it numerical way by arun umrao
Distribution of normal data understanding it numerical way by arun umraossuserd6b1fd
 
Decreasing and increasing functions by arun umrao
Decreasing and increasing functions by arun umraoDecreasing and increasing functions by arun umrao
Decreasing and increasing functions by arun umraossuserd6b1fd
 
What is meaning of epsilon and delta in limits of a function by Arun Umrao
What is meaning of epsilon and delta in limits of a function by Arun UmraoWhat is meaning of epsilon and delta in limits of a function by Arun Umrao
What is meaning of epsilon and delta in limits of a function by Arun Umraossuserd6b1fd
 
Notes for GNU Octave - Numerical Programming - for Students - 02 of 02 by aru...
Notes for GNU Octave - Numerical Programming - for Students - 02 of 02 by aru...Notes for GNU Octave - Numerical Programming - for Students - 02 of 02 by aru...
Notes for GNU Octave - Numerical Programming - for Students - 02 of 02 by aru...ssuserd6b1fd
 
Notes for GNU Octave - Numerical Programming - for Students 01 of 02 by Arun ...
Notes for GNU Octave - Numerical Programming - for Students 01 of 02 by Arun ...Notes for GNU Octave - Numerical Programming - for Students 01 of 02 by Arun ...
Notes for GNU Octave - Numerical Programming - for Students 01 of 02 by Arun ...ssuserd6b1fd
 
Notes for C++ Programming / Object Oriented C++ Programming for MCA, BCA and ...
Notes for C++ Programming / Object Oriented C++ Programming for MCA, BCA and ...Notes for C++ Programming / Object Oriented C++ Programming for MCA, BCA and ...
Notes for C++ Programming / Object Oriented C++ Programming for MCA, BCA and ...ssuserd6b1fd
 
Notes for C++ Programming / Object Oriented C++ Programming for MCA, BCA and ...
Notes for C++ Programming / Object Oriented C++ Programming for MCA, BCA and ...Notes for C++ Programming / Object Oriented C++ Programming for MCA, BCA and ...
Notes for C++ Programming / Object Oriented C++ Programming for MCA, BCA and ...ssuserd6b1fd
 
Think Like Scilab and Become a Numerical Programming Expert- Notes for Beginn...
Think Like Scilab and Become a Numerical Programming Expert- Notes for Beginn...Think Like Scilab and Become a Numerical Programming Expert- Notes for Beginn...
Think Like Scilab and Become a Numerical Programming Expert- Notes for Beginn...ssuserd6b1fd
 
Think Like Scilab and Become a Numerical Programming Expert- Notes for Beginn...
Think Like Scilab and Become a Numerical Programming Expert- Notes for Beginn...Think Like Scilab and Become a Numerical Programming Expert- Notes for Beginn...
Think Like Scilab and Become a Numerical Programming Expert- Notes for Beginn...ssuserd6b1fd
 
Notes for C Programming for MCA, BCA, B. Tech CSE, ECE and MSC (CS) 5 of 5 by...
Notes for C Programming for MCA, BCA, B. Tech CSE, ECE and MSC (CS) 5 of 5 by...Notes for C Programming for MCA, BCA, B. Tech CSE, ECE and MSC (CS) 5 of 5 by...
Notes for C Programming for MCA, BCA, B. Tech CSE, ECE and MSC (CS) 5 of 5 by...ssuserd6b1fd
 
Notes for C Programming for MCA, BCA, B. Tech CSE, ECE and MSC (CS) 4 of 5 by...
Notes for C Programming for MCA, BCA, B. Tech CSE, ECE and MSC (CS) 4 of 5 by...Notes for C Programming for MCA, BCA, B. Tech CSE, ECE and MSC (CS) 4 of 5 by...
Notes for C Programming for MCA, BCA, B. Tech CSE, ECE and MSC (CS) 4 of 5 by...ssuserd6b1fd
 
Notes for C Programming for MCA, BCA, B. Tech CSE, ECE and MSC (CS) 3 of 5 b...
Notes for C Programming for MCA, BCA, B. Tech CSE, ECE and MSC (CS) 3 of 5  b...Notes for C Programming for MCA, BCA, B. Tech CSE, ECE and MSC (CS) 3 of 5  b...
Notes for C Programming for MCA, BCA, B. Tech CSE, ECE and MSC (CS) 3 of 5 b...ssuserd6b1fd
 
Notes for C Programming for MCA, BCA, B. Tech CSE, ECE and MSC (CS) 2 of 5 by...
Notes for C Programming for MCA, BCA, B. Tech CSE, ECE and MSC (CS) 2 of 5 by...Notes for C Programming for MCA, BCA, B. Tech CSE, ECE and MSC (CS) 2 of 5 by...
Notes for C Programming for MCA, BCA, B. Tech CSE, ECE and MSC (CS) 2 of 5 by...ssuserd6b1fd
 
Notes for C Programming for MCA, BCA, B. Tech CSE, ECE and MSC (CS) 1 of 5 by...
Notes for C Programming for MCA, BCA, B. Tech CSE, ECE and MSC (CS) 1 of 5 by...Notes for C Programming for MCA, BCA, B. Tech CSE, ECE and MSC (CS) 1 of 5 by...
Notes for C Programming for MCA, BCA, B. Tech CSE, ECE and MSC (CS) 1 of 5 by...ssuserd6b1fd
 
Notes and Description for Xcos Scilab Block Simulation with Suitable Examples...
Notes and Description for Xcos Scilab Block Simulation with Suitable Examples...Notes and Description for Xcos Scilab Block Simulation with Suitable Examples...
Notes and Description for Xcos Scilab Block Simulation with Suitable Examples...ssuserd6b1fd
 
Work and Energy Notes by Arun Umrao
Work and Energy Notes by Arun UmraoWork and Energy Notes by Arun Umrao
Work and Energy Notes by Arun Umraossuserd6b1fd
 
Notes of Units, Dimensions & Errors for IIT JEE by Arun Umrao
Notes of Units, Dimensions & Errors for IIT JEE by Arun UmraoNotes of Units, Dimensions & Errors for IIT JEE by Arun Umrao
Notes of Units, Dimensions & Errors for IIT JEE by Arun Umraossuserd6b1fd
 
Physics dictionary for CBSE, ISCE, Class X Students by Arun Umrao
Physics dictionary for CBSE, ISCE, Class X Students by Arun UmraoPhysics dictionary for CBSE, ISCE, Class X Students by Arun Umrao
Physics dictionary for CBSE, ISCE, Class X Students by Arun Umraossuserd6b1fd
 

More from ssuserd6b1fd (20)

Notes for c programming for mca, bca, b. tech cse, ece and msc (cs) 1 of 5 by...
Notes for c programming for mca, bca, b. tech cse, ece and msc (cs) 1 of 5 by...Notes for c programming for mca, bca, b. tech cse, ece and msc (cs) 1 of 5 by...
Notes for c programming for mca, bca, b. tech cse, ece and msc (cs) 1 of 5 by...
 
Decreasing increasing functions by arun umrao
Decreasing increasing functions by arun umraoDecreasing increasing functions by arun umrao
Decreasing increasing functions by arun umrao
 
Distribution of normal data understanding it numerical way by arun umrao
Distribution of normal data   understanding it numerical way by arun umraoDistribution of normal data   understanding it numerical way by arun umrao
Distribution of normal data understanding it numerical way by arun umrao
 
Decreasing and increasing functions by arun umrao
Decreasing and increasing functions by arun umraoDecreasing and increasing functions by arun umrao
Decreasing and increasing functions by arun umrao
 
What is meaning of epsilon and delta in limits of a function by Arun Umrao
What is meaning of epsilon and delta in limits of a function by Arun UmraoWhat is meaning of epsilon and delta in limits of a function by Arun Umrao
What is meaning of epsilon and delta in limits of a function by Arun Umrao
 
Notes for GNU Octave - Numerical Programming - for Students - 02 of 02 by aru...
Notes for GNU Octave - Numerical Programming - for Students - 02 of 02 by aru...Notes for GNU Octave - Numerical Programming - for Students - 02 of 02 by aru...
Notes for GNU Octave - Numerical Programming - for Students - 02 of 02 by aru...
 
Notes for GNU Octave - Numerical Programming - for Students 01 of 02 by Arun ...
Notes for GNU Octave - Numerical Programming - for Students 01 of 02 by Arun ...Notes for GNU Octave - Numerical Programming - for Students 01 of 02 by Arun ...
Notes for GNU Octave - Numerical Programming - for Students 01 of 02 by Arun ...
 
Notes for C++ Programming / Object Oriented C++ Programming for MCA, BCA and ...
Notes for C++ Programming / Object Oriented C++ Programming for MCA, BCA and ...Notes for C++ Programming / Object Oriented C++ Programming for MCA, BCA and ...
Notes for C++ Programming / Object Oriented C++ Programming for MCA, BCA and ...
 
Notes for C++ Programming / Object Oriented C++ Programming for MCA, BCA and ...
Notes for C++ Programming / Object Oriented C++ Programming for MCA, BCA and ...Notes for C++ Programming / Object Oriented C++ Programming for MCA, BCA and ...
Notes for C++ Programming / Object Oriented C++ Programming for MCA, BCA and ...
 
Think Like Scilab and Become a Numerical Programming Expert- Notes for Beginn...
Think Like Scilab and Become a Numerical Programming Expert- Notes for Beginn...Think Like Scilab and Become a Numerical Programming Expert- Notes for Beginn...
Think Like Scilab and Become a Numerical Programming Expert- Notes for Beginn...
 
Think Like Scilab and Become a Numerical Programming Expert- Notes for Beginn...
Think Like Scilab and Become a Numerical Programming Expert- Notes for Beginn...Think Like Scilab and Become a Numerical Programming Expert- Notes for Beginn...
Think Like Scilab and Become a Numerical Programming Expert- Notes for Beginn...
 
Notes for C Programming for MCA, BCA, B. Tech CSE, ECE and MSC (CS) 5 of 5 by...
Notes for C Programming for MCA, BCA, B. Tech CSE, ECE and MSC (CS) 5 of 5 by...Notes for C Programming for MCA, BCA, B. Tech CSE, ECE and MSC (CS) 5 of 5 by...
Notes for C Programming for MCA, BCA, B. Tech CSE, ECE and MSC (CS) 5 of 5 by...
 
Notes for C Programming for MCA, BCA, B. Tech CSE, ECE and MSC (CS) 4 of 5 by...
Notes for C Programming for MCA, BCA, B. Tech CSE, ECE and MSC (CS) 4 of 5 by...Notes for C Programming for MCA, BCA, B. Tech CSE, ECE and MSC (CS) 4 of 5 by...
Notes for C Programming for MCA, BCA, B. Tech CSE, ECE and MSC (CS) 4 of 5 by...
 
Notes for C Programming for MCA, BCA, B. Tech CSE, ECE and MSC (CS) 3 of 5 b...
Notes for C Programming for MCA, BCA, B. Tech CSE, ECE and MSC (CS) 3 of 5  b...Notes for C Programming for MCA, BCA, B. Tech CSE, ECE and MSC (CS) 3 of 5  b...
Notes for C Programming for MCA, BCA, B. Tech CSE, ECE and MSC (CS) 3 of 5 b...
 
Notes for C Programming for MCA, BCA, B. Tech CSE, ECE and MSC (CS) 2 of 5 by...
Notes for C Programming for MCA, BCA, B. Tech CSE, ECE and MSC (CS) 2 of 5 by...Notes for C Programming for MCA, BCA, B. Tech CSE, ECE and MSC (CS) 2 of 5 by...
Notes for C Programming for MCA, BCA, B. Tech CSE, ECE and MSC (CS) 2 of 5 by...
 
Notes for C Programming for MCA, BCA, B. Tech CSE, ECE and MSC (CS) 1 of 5 by...
Notes for C Programming for MCA, BCA, B. Tech CSE, ECE and MSC (CS) 1 of 5 by...Notes for C Programming for MCA, BCA, B. Tech CSE, ECE and MSC (CS) 1 of 5 by...
Notes for C Programming for MCA, BCA, B. Tech CSE, ECE and MSC (CS) 1 of 5 by...
 
Notes and Description for Xcos Scilab Block Simulation with Suitable Examples...
Notes and Description for Xcos Scilab Block Simulation with Suitable Examples...Notes and Description for Xcos Scilab Block Simulation with Suitable Examples...
Notes and Description for Xcos Scilab Block Simulation with Suitable Examples...
 
Work and Energy Notes by Arun Umrao
Work and Energy Notes by Arun UmraoWork and Energy Notes by Arun Umrao
Work and Energy Notes by Arun Umrao
 
Notes of Units, Dimensions & Errors for IIT JEE by Arun Umrao
Notes of Units, Dimensions & Errors for IIT JEE by Arun UmraoNotes of Units, Dimensions & Errors for IIT JEE by Arun Umrao
Notes of Units, Dimensions & Errors for IIT JEE by Arun Umrao
 
Physics dictionary for CBSE, ISCE, Class X Students by Arun Umrao
Physics dictionary for CBSE, ISCE, Class X Students by Arun UmraoPhysics dictionary for CBSE, ISCE, Class X Students by Arun Umrao
Physics dictionary for CBSE, ISCE, Class X Students by Arun Umrao
 

Recently uploaded

EPANDING THE CONTENT OF AN OUTLINE using notes.pptx
EPANDING THE CONTENT OF AN OUTLINE using notes.pptxEPANDING THE CONTENT OF AN OUTLINE using notes.pptx
EPANDING THE CONTENT OF AN OUTLINE using notes.pptxRaymartEstabillo3
 
Historical philosophical, theoretical, and legal foundations of special and i...
Historical philosophical, theoretical, and legal foundations of special and i...Historical philosophical, theoretical, and legal foundations of special and i...
Historical philosophical, theoretical, and legal foundations of special and i...jaredbarbolino94
 
Capitol Tech U Doctoral Presentation - April 2024.pptx
Capitol Tech U Doctoral Presentation - April 2024.pptxCapitol Tech U Doctoral Presentation - April 2024.pptx
Capitol Tech U Doctoral Presentation - April 2024.pptxCapitolTechU
 
Like-prefer-love -hate+verb+ing & silent letters & citizenship text.pdf
Like-prefer-love -hate+verb+ing & silent letters & citizenship text.pdfLike-prefer-love -hate+verb+ing & silent letters & citizenship text.pdf
Like-prefer-love -hate+verb+ing & silent letters & citizenship text.pdfMr Bounab Samir
 
Earth Day Presentation wow hello nice great
Earth Day Presentation wow hello nice greatEarth Day Presentation wow hello nice great
Earth Day Presentation wow hello nice greatYousafMalik24
 
Computed Fields and api Depends in the Odoo 17
Computed Fields and api Depends in the Odoo 17Computed Fields and api Depends in the Odoo 17
Computed Fields and api Depends in the Odoo 17Celine George
 
Introduction to AI in Higher Education_draft.pptx
Introduction to AI in Higher Education_draft.pptxIntroduction to AI in Higher Education_draft.pptx
Introduction to AI in Higher Education_draft.pptxpboyjonauth
 
Hierarchy of management that covers different levels of management
Hierarchy of management that covers different levels of managementHierarchy of management that covers different levels of management
Hierarchy of management that covers different levels of managementmkooblal
 
Blooming Together_ Growing a Community Garden Worksheet.docx
Blooming Together_ Growing a Community Garden Worksheet.docxBlooming Together_ Growing a Community Garden Worksheet.docx
Blooming Together_ Growing a Community Garden Worksheet.docxUnboundStockton
 
Organic Name Reactions for the students and aspirants of Chemistry12th.pptx
Organic Name Reactions  for the students and aspirants of Chemistry12th.pptxOrganic Name Reactions  for the students and aspirants of Chemistry12th.pptx
Organic Name Reactions for the students and aspirants of Chemistry12th.pptxVS Mahajan Coaching Centre
 
Final demo Grade 9 for demo Plan dessert.pptx
Final demo Grade 9 for demo Plan dessert.pptxFinal demo Grade 9 for demo Plan dessert.pptx
Final demo Grade 9 for demo Plan dessert.pptxAvyJaneVismanos
 
Employee wellbeing at the workplace.pptx
Employee wellbeing at the workplace.pptxEmployee wellbeing at the workplace.pptx
Employee wellbeing at the workplace.pptxNirmalaLoungPoorunde1
 
Painted Grey Ware.pptx, PGW Culture of India
Painted Grey Ware.pptx, PGW Culture of IndiaPainted Grey Ware.pptx, PGW Culture of India
Painted Grey Ware.pptx, PGW Culture of IndiaVirag Sontakke
 
How to Configure Email Server in Odoo 17
How to Configure Email Server in Odoo 17How to Configure Email Server in Odoo 17
How to Configure Email Server in Odoo 17Celine George
 
Presiding Officer Training module 2024 lok sabha elections
Presiding Officer Training module 2024 lok sabha electionsPresiding Officer Training module 2024 lok sabha elections
Presiding Officer Training module 2024 lok sabha electionsanshu789521
 
Framing an Appropriate Research Question 6b9b26d93da94caf993c038d9efcdedb.pdf
Framing an Appropriate Research Question 6b9b26d93da94caf993c038d9efcdedb.pdfFraming an Appropriate Research Question 6b9b26d93da94caf993c038d9efcdedb.pdf
Framing an Appropriate Research Question 6b9b26d93da94caf993c038d9efcdedb.pdfUjwalaBharambe
 
ECONOMIC CONTEXT - LONG FORM TV DRAMA - PPT
ECONOMIC CONTEXT - LONG FORM TV DRAMA - PPTECONOMIC CONTEXT - LONG FORM TV DRAMA - PPT
ECONOMIC CONTEXT - LONG FORM TV DRAMA - PPTiammrhaywood
 

Recently uploaded (20)

TataKelola dan KamSiber Kecerdasan Buatan v022.pdf
TataKelola dan KamSiber Kecerdasan Buatan v022.pdfTataKelola dan KamSiber Kecerdasan Buatan v022.pdf
TataKelola dan KamSiber Kecerdasan Buatan v022.pdf
 
EPANDING THE CONTENT OF AN OUTLINE using notes.pptx
EPANDING THE CONTENT OF AN OUTLINE using notes.pptxEPANDING THE CONTENT OF AN OUTLINE using notes.pptx
EPANDING THE CONTENT OF AN OUTLINE using notes.pptx
 
ESSENTIAL of (CS/IT/IS) class 06 (database)
ESSENTIAL of (CS/IT/IS) class 06 (database)ESSENTIAL of (CS/IT/IS) class 06 (database)
ESSENTIAL of (CS/IT/IS) class 06 (database)
 
Historical philosophical, theoretical, and legal foundations of special and i...
Historical philosophical, theoretical, and legal foundations of special and i...Historical philosophical, theoretical, and legal foundations of special and i...
Historical philosophical, theoretical, and legal foundations of special and i...
 
Capitol Tech U Doctoral Presentation - April 2024.pptx
Capitol Tech U Doctoral Presentation - April 2024.pptxCapitol Tech U Doctoral Presentation - April 2024.pptx
Capitol Tech U Doctoral Presentation - April 2024.pptx
 
Like-prefer-love -hate+verb+ing & silent letters & citizenship text.pdf
Like-prefer-love -hate+verb+ing & silent letters & citizenship text.pdfLike-prefer-love -hate+verb+ing & silent letters & citizenship text.pdf
Like-prefer-love -hate+verb+ing & silent letters & citizenship text.pdf
 
Earth Day Presentation wow hello nice great
Earth Day Presentation wow hello nice greatEarth Day Presentation wow hello nice great
Earth Day Presentation wow hello nice great
 
Computed Fields and api Depends in the Odoo 17
Computed Fields and api Depends in the Odoo 17Computed Fields and api Depends in the Odoo 17
Computed Fields and api Depends in the Odoo 17
 
Introduction to AI in Higher Education_draft.pptx
Introduction to AI in Higher Education_draft.pptxIntroduction to AI in Higher Education_draft.pptx
Introduction to AI in Higher Education_draft.pptx
 
Hierarchy of management that covers different levels of management
Hierarchy of management that covers different levels of managementHierarchy of management that covers different levels of management
Hierarchy of management that covers different levels of management
 
OS-operating systems- ch04 (Threads) ...
OS-operating systems- ch04 (Threads) ...OS-operating systems- ch04 (Threads) ...
OS-operating systems- ch04 (Threads) ...
 
Blooming Together_ Growing a Community Garden Worksheet.docx
Blooming Together_ Growing a Community Garden Worksheet.docxBlooming Together_ Growing a Community Garden Worksheet.docx
Blooming Together_ Growing a Community Garden Worksheet.docx
 
Organic Name Reactions for the students and aspirants of Chemistry12th.pptx
Organic Name Reactions  for the students and aspirants of Chemistry12th.pptxOrganic Name Reactions  for the students and aspirants of Chemistry12th.pptx
Organic Name Reactions for the students and aspirants of Chemistry12th.pptx
 
Final demo Grade 9 for demo Plan dessert.pptx
Final demo Grade 9 for demo Plan dessert.pptxFinal demo Grade 9 for demo Plan dessert.pptx
Final demo Grade 9 for demo Plan dessert.pptx
 
Employee wellbeing at the workplace.pptx
Employee wellbeing at the workplace.pptxEmployee wellbeing at the workplace.pptx
Employee wellbeing at the workplace.pptx
 
Painted Grey Ware.pptx, PGW Culture of India
Painted Grey Ware.pptx, PGW Culture of IndiaPainted Grey Ware.pptx, PGW Culture of India
Painted Grey Ware.pptx, PGW Culture of India
 
How to Configure Email Server in Odoo 17
How to Configure Email Server in Odoo 17How to Configure Email Server in Odoo 17
How to Configure Email Server in Odoo 17
 
Presiding Officer Training module 2024 lok sabha elections
Presiding Officer Training module 2024 lok sabha electionsPresiding Officer Training module 2024 lok sabha elections
Presiding Officer Training module 2024 lok sabha elections
 
Framing an Appropriate Research Question 6b9b26d93da94caf993c038d9efcdedb.pdf
Framing an Appropriate Research Question 6b9b26d93da94caf993c038d9efcdedb.pdfFraming an Appropriate Research Question 6b9b26d93da94caf993c038d9efcdedb.pdf
Framing an Appropriate Research Question 6b9b26d93da94caf993c038d9efcdedb.pdf
 
ECONOMIC CONTEXT - LONG FORM TV DRAMA - PPT
ECONOMIC CONTEXT - LONG FORM TV DRAMA - PPTECONOMIC CONTEXT - LONG FORM TV DRAMA - PPT
ECONOMIC CONTEXT - LONG FORM TV DRAMA - PPT
 

Maxima & Minima of Functions - Differential Calculus by Arun Umrao

  • 1. 1 MAXIMA & MINIMA A SHORT NOTES Arun Umrao https://sites.google.com/view/arunumrao DRAFT COPY - GPL LICENSING
  • 2. 2 Maxima & Minima Contents 1 Extrema 3 1.1 Maxima & Minima . . . . . . . . . . . . . . . . . . . . . . . . 3 1.1.1 Maximum & Minimum Function . . . . . . . . . . . . . 3 1.2 Some Definitions . . . . . . . . . . . . . . . . . . . . . . . . . 4 1.2.1 Maxima & Minima Points . . . . . . . . . . . . . . . . 5 1.2.2 Critical Points . . . . . . . . . . . . . . . . . . . . . . . 5 1.2.3 Stationary Points . . . . . . . . . . . . . . . . . . . . . 6 1.2.4 Point of Inflection . . . . . . . . . . . . . . . . . . . . . 6 1.2.5 Extreme Value Theorem . . . . . . . . . . . . . . . . . 8 1.2.6 Global Minimum & Global Maximum . . . . . . . . . . 8 1.2.7 Local Minimum & Local Maximum . . . . . . . . . . . 9 1.2.8 Saddle Point . . . . . . . . . . . . . . . . . . . . . . . . 10 1.3 Extremum Test . . . . . . . . . . . . . . . . . . . . . . . . . . 11 1.4 Method of Finding Extremum . . . . . . . . . . . . . . . . . . 11 1.4.1 First Derivative Test . . . . . . . . . . . . . . . . . . . 11 1.4.2 Second Derivative Test . . . . . . . . . . . . . . . . . . 12 1.4.3 Why Second Derivative Required . . . . . . . . . . . . 13 1.5 Extremum in Two-Dimensional Function . . . . . . . . . . . . 15 1.6 Applied Problems . . . . . . . . . . . . . . . . . . . . . . . . . 16 1.6.1 Hypothetical Extrema . . . . . . . . . . . . . . . . . . 29
  • 3. 1.1. MAXIMA & MINIMA 3 1Maxima & Minima 1.1 Maxima & Minima Maxima and Minima are points where function reaches its maximum and minimum value respectively. These points are also called extrema. Extrema are of two types: (i) Global extrema (Absolute extrema) and (ii) Local ex- trema (Relative extrema). Global extrema are those points whose value is maximum or minimum on the entire range of the function. Local extrema are the largest or smallest values of the function in the immediate vicinity. At extrema, the slope of a function in graph is necessarily zero. Extrema are also called stationary points or turning points. 1.1.1 Maximum & Minimum Function Certain functions have maximum and minimum values at their certain points. At other points function value oscillates between these two extreme values. Illustrated Example Assume sine function y = a sin(x). The domain of argument of sine varies from 0 to 2nπ. Range of sine function lies between -1 to +1 irrespective of value of argument. For Maximum y y shall be maximum if sin(x) is maximum, i.e. sin(x) = 1 or x = π/2. Now the y is y = a. For Minimum y y shall be minimum if sin(x) is minimum, i.e. sin(x) = −1 or x = −π/2. Now the y is y = −a. 1 2 3 4 5 6 x y a −a
  • 4. 4 Maxima & Minima Solved Problem 1.1 Find the maximum and minimum value of y = a sin2 (x) + b cos2 (x) and a > b. Solution The given trigonometric function is y = a sin2 (x)+b cos2 (x) and it is periodic function. General value of arguments of sine and cosine varies from 0 to 2nπ. Range of sine and cosine lies between -1 to +1. Hence y shall continuously changes between its maximum and minimum values. Changing the function for one trigonometric form y = a sin2 (x) + b 1 − sin2 (x) Simplifying it for sine y = b + (a − b) sin2 (x) For Maximum y y shall be maximum if sin2 (x) is maximum, i.e. sin2 (x) = 1 or sin(x) = ±1. Here taking only +1, x = π/2. Now the y is y = b + (a − b) = a For Minimum y y shall be minimum if sin2 (x) is minimum, i.e. sin2 (x) = −1 or sin(x) = ± √ −1. It is imaginary value for sine, hence not acceptable. So for only acceptable values sin2 (x) = 0, i.e. x = 0. Now the y is y = b + (a − b) × 0 = b 1 2 3 x y a b x y a b 1.2 Some Definitions Following definitions are useful in finding the maximum or minimum value of a function.
  • 5. 1.2. SOME DEFINITIONS 5 1.2.1 Maxima Minima Points Maxima and minima points on combining known as extrema points. At extrema points curves changes it direction. There may one or more than one extrema points for a given function. x y x = ay2 (0, 0) b (a) x y x = a − y2 (0, 2) b (b) Figure 1.1: Function plot. From figure 1.1(1) point (0, 0) is the point at deepest position. This point is known as the minima point in the given range of the function f(x). Similarly from figure 1.1(2), point (0, 2) is the point at maximum height. It is known as maxima point for given range of the function f(x). 1.2.2 Critical Points A single variable function f(x) has critical points at all points x0 where f ′ (x0) = 0 or f(x) is not differentiable at the given point. x f(x) f(x) = 2x2 (0, 0) b f ′ (x) = 0 (a) x f(x) f(x) = 4 − x2 (0, 2) b f ′ (x) = 0 (b) Figure 1.2:
  • 6. 6 Maxima Minima In figure 1.2(1) the tangent at lowest depth point is parallel to the positive x axis. Similarly in figure 1.2(2) tangent at maximum height of the curve is parallel to the positive x axis. The points at which tangent is parallel to the x axis are known as critical points or stationary points. At critical points slope of tangent f ′ (x) is zero. Multi-variable function f(x, y) has critical points where either the gradient df(x, y) is zero or partial derivative of function is not defined at that point, ie ∂f(x, y)/∂x or ∂f(x, y)/∂y is undefined. 1.2.3 Stationary Points Points at which the derivative of a function f(x) vanishes. For example, at point x0 if first derivative of function is zero, i.e. f ′ (x0) = 0 Then point x0 is known as stationary point. This stationary point may be minimum, maximum or inflection point. Value of function at stationary point is called stationary value. Solved Problem 1.2 Find the stationary points of function f(x) = 2x − x2 . Solution The stationary points of the function f(x) is given by f ′ (x) = 0, i.e. f ′ (x) = 2 − 2x = 0 ⇒ x = 1 The stationary point of the given function is x = 1. 1.2.4 Point of Inflection It is a point on a curve at which the curvature or concavity changes sign from plus to minus or vice-versa. At reflection point, curve changes from being concave (concave downward) to convex (concave upward), or vice versa. If f(x) is k times continuously differentiable in a certain neighbourhood of a point x with k odd and k ≥ 3, while f(n) (x0) = 0 for n = 2, . . ., k1 and f(n) (x0) 6= 0 then f(x) has a point of inflection at x = x0.
  • 7. 1.2. SOME DEFINITIONS 7 1 −1 1 −1 x f(x) b f(x) = x3 f ′ (x) = 3x2 f ′′ (x) = 6x Above graph is drawn for f(x) = x3 . The point x at which slope of curve is parallel to horizontal axis, is found by using f ′ (x) = 0 It gives x = 0. Now again derivating f ′ (x), we get f ′ (x) = 6x f ′ (x) is zero at x = 0, hence x = 0 is point of inflection of given function f(x). In the following figure, function g(x) = x3 − 3x2 + 2x is plotted. Its derivative is g ′ (x) = 3x2 − 6x + 2. 1 −1 1 2 −1 x g(x) b g(x) g ′ (x) g ′′ (x) The point of inflection shall be where, there g ′′ (x) = 0. So, g ′ (x) = 0 = 6x − 6 It gives, x = 1. This is inflection point for the given function g(x). Note that, inflection point may or may not be a root point, i.e. zeros of the function. For example, take y = x3 − 3x2 − 144x + 432
  • 8. 8 Maxima Minima Its roots are x = −12, x = 3 and x = 12. The point of inflection will be found by taking y ′′ = 0. So, 6x − 6 = 0 ⇒ x = 1 which is not root of the given function y. 1.2.5 Extreme Value Theorem If a function h(x) is continuous on a closed interval [p, q], then h(x) has both maximum and minimum on closed interval [p, q]. If h(x) has an extremum on an open interval (p, q) then extremum occurs at a critical point. 1.2.6 Global Minimum Global Maximum The overall largest value of a set, function, etc., over its entire range is called global maximum. While smallest value of a set, function, etc., over its entire range is called global minimum. It is impossible to construct an algorithm that will find a global maximum or global minimum for an arbitrary function. We can check whether function has global extrema or not by just putting x → ±∞. For example, y = x2 − 2x + 4 global maxima as lim x→−∞ y = (−∞)2 − 2 × −∞ + 4 = ∞ and lim x→+∞ y = ∞(∞ − 2) + 4 = ∞ Thus function is not converging when x → ±∞. Using these tests, we can expect local extrema. 1. When limx→−∞ y = limx→+∞ y = ±∞, i.e. both have same values with then function may have odd numbers of extrema. This happens when highest degree of function is even.
  • 9. 1.2. SOME DEFINITIONS 9 x y x → −∞ x → ∞ x y x → −∞ x → ∞ 2. When limx→−∞ y = −∞ and limx→+∞ y = +∞ or vice-versa then function may have even numbers of extrema. This happens when highest degree of function is odd. x y x → −∞ x → ∞ x y x → −∞ x → ∞ 1.2.7 Local Minimum Local Maximum It is also known as relative minimum and relative maximum. As- sume a function f(x), whose successive derivatives at point x0 are zero. i.e. f ′ (x0) = 0, f ′′ (x0) = 0, . . . . . . , f(n) (x0) = 0 But f(n+1) (x0) 6= 0 Now, f(x) has a local maximum at x0 if n is odd and f(n+1) (x0) 0. Sim- ilarly, f(x) is local minima at x0 if n is odd and f(n+1) (x0) 0. The (n)th derivative of function is zero at point x0 but (n)th derivative of function is not zero at point x0 hence it gives the maximum and minimum point. To find the saddle point we have to find the maximum derivatives of function that gives zero value at point x0 and also check is n odd or even. In this case, highest derivative that gives zero value at x0 has derivative power n. There is a saddle point at x0 if n is even. Bounding Values If there are extrema points within closed range [a, b], then the extreme values (positive or negative) of the function are also known
  • 10. 10 Maxima Minima as bounding values of the function. For example, suppose function f(x) = x − x2 which is continuous and differentiable at every point defined within (0, 1). The function has a extrema at f ′ (x) = 1 − 2x = 0 ⇒ x = 0.5 Now the function value at x = 0.5 is f(0.5) = 0.25 Again second derivative f ′′ (0.5) = −2 is negative value. Hence the extrema is also local maxima. Now the roots of function are x − x2 = 0 ⇒ x = 0, 1 Function value at these two points are f(0) = 0 f(1) = 0 Hence the function value ranges between (0, 0.25) ie in other words the function is bounded between 0 to 0.25 (upper bound by 0.25 and lower bound by 0). 1.2.8 Saddle Point A point on a function f(x) which is a stationary point but not an extremum. To clarify the point assume an function f(x) = 3x3 . Saddle point is mostly visible in one-dimensional functions with one term. First derivative of the function is f ′ (x) = 9x2 . Second derivative of the function is f ′′ (x) = 18x. And third derivative is f ′′′ (x) = 18. The function has a saddle point at x0 = 0 by extremum test as1 f ′′ (x0) = 0; f ′′′ (x0) = 18 1 3rd derivative has non zero value at point x0 while 2nd derivative has zero value at x0. 2 power in second derivative is an even number hence there is a saddle point at x0.
  • 11. 1.3. EXTREMUM TEST 11 Where, third derivative is not zero at point x0. 1.3 Extremum Test To find the extrema of a function f(x) at all points x0, first derivative of func- tion f(x) is obtained. As the slope must be zero at stationary points hence the turning points can be calculated by equating first derivative to zero, ie f ′ (x) = 0. This gives all the values of points x0. To classify a stationary point extremum test or second derivative test is carried out. If second deriva- tive of the function f(x) at extremum points is positive (f ′′ (x0) 0) then gradient is increasing and stationary point is minimum. If second derivative of the function f(x) is negative (f ′′ (x0) 0) then gradient is decreasing and stationary point is maximum. If second derivative is zero at stationary point (f ′′ (x0) = 0) then it is a point of inflection or it could be an extremum. To find the extrema we differentiate the first derivative until we would not find the non zero derivative at that stationary point. 1.4 Method of Finding Extremum There are two methods of finding extremum. (i) First derivative test and (ii) second derivative test. 1.4.1 First Derivative Test Suppose a function f(x) that is continuous at a stationary point x0. First derivative of the function f(x) represents the slope of tangent on the function at the point ‘x’. For maxima or minima at any point on the function graph, following rules are observed. 1. If f ′ (xl) 0 on an open interval extending left from x0 and f ′ (xr) 0 on an open interval extending right from x0, then f(x) has a local maximum at x0. It is also possibility that there is an global maximum.
  • 12. 12 Maxima Minima x y b b b xl x0 xr f ′ (xl) 0 f ′ (x0) = 0 f ′ (xr) 0 +θ −θ 2. If f ′ (xl) 0 on an open interval extending left from x0 and f ′ (xr) 0 on an open interval extending right from x0, then f(x) has a local minimum at x0. It is also possibility that there is an global minimum. x y b b b xl x0 xr f ′ (xl) 0 f ′ (x0) = 0 f ′ (xr) 0 −θ +θ 3. If f ′ (x) has the same sign on an open interval extending left from x0 and on an open interval extending right from x0, then f(x) has a point of inflection at x0. 1.4.2 Second Derivative Test Suppose a function f(x) is continuous and twice differentiable at stationary point x0. There are two cases of the f ′′ (x) at point x0. 1. If f ′′ (x0) 0, then slope of tangent on function f ′ (x0) is upward and the function f(x0) has a relative minima at stationary point x0. 2. If f ′′ (x0) 0, then slope of tangent on function f ′ (x0) is downward and the function f(x0) has a relative maxima at stationary point x0.
  • 13. 1.4. METHOD OF FINDING EXTREMUM 13 x y b b b xl x0 xr f ′ (xl) f ′ (x0) f ′ (xr) (a) x y b b b xl x0 xr f ′′ (xl) f ′′ (x0) f ′′ (xr) (b) −θ (c) Figure 1.3: Figure (a) graphed presentation of first derivative of sin(x), (b) graphed presentation of second derivative of sin(x) and (c) slope of second derivative of sin(x) at point x0. 1.4.3 Why Second Derivative Required Take a function f(x) whose curve is shown in the following figure. It has four inversion points at x = −2, x = −1, x = 0, x = 1 and x = 2. First derivative of function f(x) is plotted too. f ′ (x) is plot of numeric values of the tangents on functions f(x). It means, f ′ (x) should have zero numeric value where function f(x) has function inversion. This is why, to get the inversion points of function f(x), first derivative of function should be equal to zero. It gives all inversion points but it does not tell about the point that whether it is maxima or it is minima. The first derivative of function, i.e. f ′ (x) clearly tells about the nature of function. When f ′ (x) 0, function is increasing and when f ′ (x) 0 function is decreasing. x f(x) f f ′ x = 1.58 x = 0 x = −1.58 (a) x f(x) f f ′ f ′′ x = 1.58 x = 0 x = −1.58 (b) When function changes from decreasing to increasing, the numeric value of f ′ (x) changes from negative value to positive value. It means, f ′ (x) has zero crossing at point of inversion and function plot has slop too. If this slope is positive then f ′′ (x) should be positive value. Similarly, when function changes from increasing to decreasing, the numeric value of f ′ (x) changes
  • 14. 14 Maxima Minima from positive value to negative value. It means, f ′ (x) has zero crossing at point of inversion and function plot has slop too. If this slope is negative then f ′′ (x) should be negative value. Hence, where f ′′ (x) is negative, then at that point function has maxima and where f ′′ (x) is positive, then at that point function has minima. This is the reason, that second derivative is required to verify the maxima and minima point. Solved Problem 1.3 A company produces auto parts and earns income from sale of these parts. The earning from sale of auto parts is based on the labour (l) employed and capital (c) invested by the company. Earning function is I = 2l + 4c − 3l2 − 2c2 + 9lc Find the quantity of labour employed and amount of capital invested by the company to earn the maximum income. Solution The variation in the income of the company when there is variation in labour employed or capital invested is maximum or minimum when dI/dl = 0 and dI/dc = 0. So, dI dl = d dl 2l + 4c − 3l2 − 2c2 + 9lc = 2 − 6l + 9c = 0 And dI dc = d dc 2l + 4c − 3l2 − 2c2 + 9lc = 4 − 4c + 9l = 0 1 0.5 1.0 1.5 2.0 2.5 3.0 3.5 4.0 4.5 5.0 l c I l c Two relations of labour and capital are 6l − 9c = 2; 9l − 4c = 4 On solving these equations, we have l = 68/171 and c = 2/19. Again, we check whether income is maximum at these values of labour employed and
  • 15. 1.5. EXTREMUM IN TWO-DIMENSIONAL FUNCTION 15 capital invested. So, we shall find d2 I/dl2 and d2 I/dc2 . So, d2 I dl2 = −6; d2 I dc2 = −4 As both are negative, hence, income shall be maximum at this labour em- ployed and capital invested. 1.5 Extremum in Two-Dimensional Function If f(x, y) is a two-dimensional function that has a relative extremum at a point (x0, y0) and has continuous partial derivatives at this point, then ∂f(x0, y0) ∂x = 0 and ∂f(x0, y0) ∂y = 0 The second partial derivatives test classifies the point as a local maximum or relative minimum. The second derivatives of the function is D then D = fxxfyy − f2 xy Now 1. If D 0 and fxx(x0, y0) 0, the point is a relative minimum. 2. If D 0 and fxx(x0, y0) 0, the point is a relative maximum. 3. If D 0, the point is saddle point. 4. If D = 0, higher order tests must be used. Note that, if we have not given the points x0 and y0 then we can find these points where extremum may exist by solving the equations obtained by using relation ∂f/∂x = 0 and ∂f/∂y = 0.
  • 16. 16 Maxima Minima 1.6 Applied Problems Solved Problem 1.4 Find the maximum value of y = x3 − 9x. Solution The given function y is zero when x = 0. To get the critical points where tangent on the function is parallel to abscissa, we derivative the function with respect to variable ‘x’. Now dy dx = 3x2 − 9 0 = 3x2 − 9 On solving, the critical points are x = ± √ 3. x f(x) f ( x ) = x 3 − 9 x b f ′ (x) = 0 b f ′ (x) = 0 (a) x f(x) f ( x ) = x 3 − 9 x b f(− √ 3) b (− √ 3, 0) b f( √ 3) b ( √ 3, 0) (b) Figure 1.4: From figure 1.4(b), function is continuously decreasing when x ≤ − √ 3 and continuously increasing when x ≥ √ 3. In the domain of − √ 3 ≤ x ≤ √ 3 function is decreasing. To verify that these points are at minima or maxima location, second derivative value of the function y is obtained at these critical points. d2 y dx2 = 6x At x = − √ 3, second derivative is negative while at x = √ 3, second derivative is positive. Now the maxima exists at x = − √ 3 and maximum function value is y− √ 3 = −3 √ 3 + 9 √ 3 = 6 √ 3 This is the local maximum value of the function. But from the figure 1.4(b) it is clear that the minimum value of the function is −∞ and maximum value of the function is +∞ globally.
  • 17. 1.6. APPLIED PROBLEMS 17 Solved Problem 1.5 Find the maximum value of f(x) = 6x − x2 . Solution The given function is f(x) = 6x − x2 . To find the maximum value of the function, we must knew the point where function is maximum. For this, f ′ (x) = 0. 6 − 2x = 0 ⇒ x = 3 There is only one extrema point. x f(x) f ( x ) = 6 x − x 2 b f ′ (x) = 0 (a) x f(x) f ( x ) = 6 x − x 2 b f(3) b (3, 0) (b) At x = 3 function shall be either minima or maxima. To check the extremum of the function, we shall find f ′′ (x)
  • 18.
  • 19. x=3 . Now, f ′′ (3) = −2 0 Hence function is maxima at x = 3. Now the function value at x = 3 is f(3) = 6 × 3 − 32 = 9 This is result. Note that, function has only one critical point where extrema function is maximum. Hence there is no definite minimum function value. Solved Problem 1.6 Find the minimum value of f(x) = x2 − 4x + 4. Solution The given function is f(x) = x2 −4x+ 4. To find the minimum value of the function, we must knew the point where function is minimum. For this, f ′ (x) = 0. 2x − 4 = 0 ⇒ x = 2 There is only one extrema point.
  • 20. 18 Maxima Minima x f(x) b f ′ (x) = 0 (a) x f(x) b f(2) b (2, 0) (b) At x = 2 function shall be either minima or maxima. To check the extremum of the function, we shall find f ′′ (x)
  • 21.
  • 22. x=2 . Now, f ′′ (2) = 2 0 Hence function is minima at x = 2. Now the function value at x = 2 is f(2) = 22 − 4 × 2 + 4 = 0 This is result. Note that, function has only one critical point where extrema function is minimum. Hence there is no definite maximum function value. Solved Problem 1.7 Find the minimum value of f(x) = x 4 − x3 5 . Solution The given function is f(x) = x 4 − x3 5 This function has extremum where f; (x) = 0. So, 1 4 − 3x2 5 = 0 ⇒ 5 − 12x2 = 0 It gives x = −0.65 and x = +0.65. This function has two extrema. Hence we shall test whether function minima or function maxima exits. x f(x) b f ′ (x) = 0 b f ′ (x) = 0 (a) x f(x) b f(0.65) b (0.65, 0) b f(−0.65) b (−0.65, 0) (b)
  • 23. 1.6. APPLIED PROBLEMS 19 To test the extrema, we shall find the f ′′ (x) at x = ±0.65. So, f ′′ (x) = − 6x 5 f ′′ (x) is negative at x = 0.65, hence function maxima is at x = 0.65. f ′′ (x) is positive at x = −0.65. hence function is minima at x = −0.65. The minimum function value is f(−0.65) = −0.65 4 − (−0.65)3 5 = 0.107575 and the function maximum value is f(0.65) = 0.65 4 − (0.65)3 5 = 0.107575 Hence, function minimum value is −0.10 approximately. Solved Problem 1.8 Find the maximum value of f(x) = 2 √ x − 2x + 3. Solution To get the critical points of the function, we should put f ′ (x) = 0. So, 2 × 1 2 × 1 √ x − 2 = 0 This give √ x = 0.5 ⇒ x = 0.25 x f(x) b f ′ (x) = 0 f = 2 √ x − 2x + 3 (a) x f(x) b f(1/4) b (1/4, 0) f = 2 √ x − 2x + 3 (b) This has only one x = 0.25, where one extrema exists. To check whether function is maximum or minimum at this point, we shall find f ′′ (0.25). So, f ′′ (x) = − 1 2 × 1 3 √ x 0; ∀x ∈ R+
  • 24. 20 Maxima Minima At x = 0.25, f ′′ (0.25) is negative, hence function is maxima at x = 0.25. The function value at this point is given by f(0.25) = 2 × √ 0.25 − 2 × 0.25 + 3 = 3.5 This is desired result. Solved Problem 1.9 Find the maximum value of f(x) = 6x − x2 . Solution To get the critical points of the function, we should put f ′ (x) = 0. So, 6 − 2x = 0 ⇒ x = 3 x f(x) f(x) = 6x − x2 b f ′ (x) = 0 (a) x f(x) f(x) = 6x − x2 b f(3) b (3, 0) (b) This has only one x = 3, where one extrema exists. To check whether function is maximum or minimum at this point, we shall find f ′′ (3). So, f ′′ (x) = −2 0; ∀x At x = 3, f ′′ (3) is negative, hence function is maxima at x = 3. The function value at this point is given by f(3) = 6 × 3 − 32 = 9 This is desired result. Solved Problem 1.10 Find the minimum value of f(x) = cos(3x) in range of [0, π/2]. Solution The given function is f(x) = cos(3x). The tangents on the function at its extrema will be there where f ′ (x) = 0. So, −3 sin(3x) = 0 ⇒ sin(3x) = 0
  • 25. 1.6. APPLIED PROBLEMS 21 Note that, function is a periodic function, hence it shall have infinite numbers of extrema and corresponding critical points. But we are taking here only local extrema, i.e. maximum and minimum values within definite domain. x f(x) f(x) = cos(3x) b b f ′ (x) = 0 f ′ (x) = 0 (a) x f(x) f(x) = cos(3x) b b f (0) f (1.046) b b (0, 0) (1.046, 0) (b) Now, 3x = nπ or x = nπ/3. We have to find the minimum value in x ∈ [0, π/2]. For this domain, only accepted critical points2 are found when n = 0 and n = 1. So, x is 0 and π/3. To check where, function is minimum, we have to find f ′′ (x) 0. Now f ′′ (x) = −9 cos(3x) f ′′ (x) is positive when cos(3x) is negative, i.e. π/2 ≤ 3x ≤ π or π/6 ≤ x ≤ π/3. Note that we are talking about cos(3x) of f ′′ (x) not about the cos(3x) of f(x). So, only acceptable value for minima from x = 0 and x = π/3 is x = π/3. Now minimum function value is f(π/3) = cos 3 × π 3 = −1 This is desired result. Solved Problem 1.11 Find the maximum value of y = sin(x) + tan(x) in range of (−2π, 2π). Solution The given function is y = sin(x) + tan(x). To get the critical points, we should put y ′ (x) = 0. Now, cos(x) + sec2 (x) = 0 On solving it, we have cos3 (x) = −1 2 Roots are those points where function is zero. Critical points are those points where tangent on the function is zero.
  • 26. 22 Maxima Minima This has three roots, in which only one is real. The real root is cos(x) = −1. On solving it, we have cos(x) = cos(2n + 1)π ⇒ x = (2n + 1)π Here, n is integer. The values of x withing (−2π, 2π) is −π and π only when n = −1 and n = 0. These are only points where extrema exist. x y(x) y(x) = sin(x) + tan(x) b b y ′ (x) = 0 y ′ (x) = 0 (a) x y(x) y(x) = sin(x) + tan(x) f (−π) f (π) b b b b (−π, 0) (π, 0) (b) To check the nature of extrema, we shall get the value of y ′′ (x) at x = −π and x = π. Hence y ′′ (x) = 2 sec2 (x) × tan(x) − sin(x) At x = −π y ′′ (−π) = 2 sec2 (−π) × tan(−π) − sin(−π) = 0 At x = π y ′′ (π) = 2 sec2 (π) × tan(π) − sin(π) = 0 At both critical points, y ′′ (x) is zero. Hence function has reflection at points x = ±π. Solved Problem 1.12 Find the range of θ for sine function of unit amplitude so that the bounded rectangular area is maximum. Solution θ y b θ b f(θ) b π − θ b f(θ) θ y
  • 27. 1.6. APPLIED PROBLEMS 23 As per the symmetry of the sine function of unit amplitude, shape of maximum area will be in rectangular shape as shown in above figure. Area of this shape is A = f(θ) × (π − 2θ) On derivating it with respect to θ and equating it to zero, it gives tan(θ) + θ − π 2 = 0 The solution of this relation gives the critical value of θ within the limit of 0 θ π is θ = 0.7104. Second derivative of the area is A′′ = − [cos θ − (π − 2θ) sin θ − 2 cos θ] It is negative when θ is substituted. Area will be maximum at the criti- cal value of θ = 0.7104. Maximum area is bounded between the limits of 0.7104 ≤ θ ≤ 2.4311. Solved Problem 1.13 Find the minimum and maximum value of f(x) = sin2 (x) in range of [0, π/2]. Solution The critical points of the function are found by putting f ′ (x) = 0. So, 2 sin(x) × cos(x) = 0 ⇒ sin(2x) = 0 This gives sin(2x) = sin(nπ) ⇒ x = n π 2 The values of x lies in [0, π/2] are x = 0 and x = π/2. x f(x) f(x) = sin2 (x) b b f ′ (x) = 0 f ′ (x) = 0 (a) x f(x) f(x) = sin2 (x) b b f π 2 f (0) b b π 2 , 0 (0, 0) (b) The extrema of function at these critical points are either maxima or minima. To check this, we have to find sign of f ′′ (x). f ′′ (x) = 2 cos(2x)
  • 28. 24 Maxima Minima At x = 0, f ′′ (0) = 2 0, hence function has minima at x = 0. Similarly at x = π/2, f ′′ (π/2) = −2 0, hence function has maxima at x = π/2. The minima and maxima values are respectively f(0) = [sin(0)]2 = 0 and f(π/2) = [sin(π/2)]2 = 1 This is desired result. Solved Problem 1.14 Find the minimum and maximum value of f(x) = sin(x2 ) in range of [0, π/2]. Solution To get the extrema of the function, we should put f ′ (x) = 0. Now, it gives cos(x2 ) × 2x = 0 This gives x = 0 and cos(x2 ) = cos((2n + 1)π/2). Or x2 = (2n + 1)π 2 This gives, x = ± r (2n + 1)π 2 x f(x) f(x) = sin(x2 ) b b b f ′ (x) = 0 f ′ (x) = 0 f ′ (x) = 0 (a) x f(x) f(x) = sin(x2 ) b b b f − q π 2 f (0) f q π 2 b b b − q π 2 , 0 (0, 0) q π 2 , 0 (b) Values of x in the domain [0, π/2] are x = 0 and at n = 0. x = + p π/2. x = − p π/2 is not acceptable as it is out of domain. To check where function is maximum and where function is minimum, we have to check sign of f ′′ (x). So f ′′ (x) = cos(x2 ) × 2 + 2x × − sin(x2 ) × 2x Or f ′′ (x) = 2 cos(x2 ) − 4x2 sin(x2 )
  • 29. 1.6. APPLIED PROBLEMS 25 This value at x = 0 and x = p π/2 are f ′′ (0) = 2 cos(02 ) − 4 × 02 × sin(02 ) = 2 0 f ′′ ( p π/2) = 2 cos(π/2) − 4 × π 2 × sin(π/2) = −2π 0 Hence function is maximum at x = p π/2 and minimum at x = 0. So maximum and minimum function values are respectively: f r π 2 = sin π 2 = 1 f(0) = sin(0) = 0 Solved Problem 1.15 Is it possible to find the maximum or minimum values of the function f(x) = tan(2x) in range of [0, π/2]? Solution If tangents on the curve are not parallel to horizontal axis in the given domain [0, π/2], then function is not changing its direction from increasing to decreasing or vice-versa in the given domain [0, π/2]. If there is a point within the domain [0, π/2] at which tangent is parallel to x-axis, then we must put f ′ (x) = 0 and find the critical points. If there are nay any critical point within the domain [0, π/2], then there is no tangent that is parallel to horizontal axis and exists within the domain [0, π/2]. x f(x) f(x) = tan(2x) b b (0, 0) π 2 , 0 So, f ′ (x) = sec2 (2x) × 2 = 0 This gives sec2 (2x) = 0 ⇒ 1 + tan2 (2x) = 0
  • 30. 26 Maxima Minima Or tan2 (2x) = −1 ⇒ tan(2x) = √ −1 = i Now, there are no points of x within given domain for which tangent satisfy above condition. Hence tangents on the given function are not parallel to the x-axis. Hence it is not possible to find the maximum or minimum value of the given function at given domain. Solved Problem 1.16 Is function f(x) = tan(2x) is strictly increasing in range of [0, π/2]? Solution For any two points a, b ∈ I, if a function f(x) satisfy the relation f(b) f(a), then it is said to be strictly increasing function for all b a. If function has f(b) ≥ f(a) for any two values of a and b then we said that function is nonstrictly increasing. So, for the given function, we first check whether there is minima or maxima of the function in [0, π/2]. If there is so, two points either side of the extrema shall give equal values of function. So, f ′ (x) = 2 sec2 (2x) = 0 The range of secant is [1, ∞). So, for any value of x, there is never zero value of secant. So, no point exits in [0, π/2] at which there is extrema on the given function f(x). Now, for two extreme values, x = 0 and x = π/2, function values are f(0) = tan(0) = 0; f(π/2) = tan(2 × π/2) = tan(π) = 0 Here, two values f(0) and f(π/2) are equal, hence function is not strictly increasing in the given domain x ∈ [0, π/2]. Solved Problem 1.17 Income of a company is represented by I = x3 −cx+3. Find the maximum income of the company when c = 3. Solution The income of a company is given by I = x3 − cx + 3 This income curve shall be maximum or minimum, where I ′ = 0. So, 3x2 − c = 0 ⇒ ±x = r c 3
  • 31. 1.6. APPLIED PROBLEMS 27 The income shall be maximum at x if I ′′ (x) 0. So, I ′′ (x) = 6x This is negative at x = −
  • 32.
  • 33. pc 3
  • 34.
  • 35. , hence income is maximum at x = −
  • 36.
  • 37. pc 3
  • 38.
  • 39. . Substituting c = 3, we have, x = −1 Now, maximum income of the company is Imax = (−1)3 − 3 × −1 + 3 = 5 This is maximum income of the company. Solved Problem 1.18 Marks of a student is represented by M = 9x2 −3x+5. Find the maximum marks of the student. Solution The maximum marks is given by M = 9x2 − 3x + 5 This mark curve shall be maximum or minimum, where M ′ = 0. So, 18x − 3 = 0 ⇒ x = 1 6 The marks shall be maximum at x if M ′′ (x) 0. So, M ′′ (x) = 18 0 This is positive at x = 1/6 and it is minima condition. Hence there is no upper limit of maximum marks. So, this marks curve tells that student has no definite maximum marks. Solved Problem 1.19 Sum of two arbitrary numbers is 24. Find the minimum value of product of these two arbitrary numbers if product is given by p = xy2 . Solution The two numbers are x and y. According the question, x+ y = 24. The product function is p = xy2 = (24 − y)y2 This product curve shall be maximum or minimum, where p ′ = 0. So, 48y − 3y2 = 0 ⇒ 3y(16 − y) = 0
  • 40. 28 Maxima Minima This gives, y = 0 or y = 16. y = 0 is not acceptable as it make product zero. Other number is y = 16. We shall check whether product is maximum for y = 16. For this p ′′ (y) 0. So, p ′′ (y) = 48 − 6y This is negative, at y = 16, hence product shall be maximum at y = 16. Now the other number is x = 24 − 16 = 8. The product is p = 8 × 162 = 2048 This is desired result. Solved Problem 1.20 Velocity of a moving particle in horizontal plane is represented by v = x3 − 4x. Here x is an arbitrary parameter. Find the maximum velocity of the particle. Solution The velocity function is v = x3 − 4x This velocity curve shall be maximum or minimum, where v ′ = 0. So, 3x2 − 4 = 0 ⇒ ±x = r 4 3 We shall check where velocity is maximum. For this v ′′ (x) 0. So, v ′′ (x) = 6x v ′′ (x) is negative at x = − q 4 3 , hence velocity shall be maximum at this point. Now, velocity is v = − r 4 3 !3 − 4 × − r 4 3 = 16 3 √ 3 This is desired result.
  • 41. 1.6. APPLIED PROBLEMS 29 1.6.1 Hypothetical Extrema There are some hypothetical extrema exist in a function. For example, a time function h = t3 − 6t − 8 has an exremum point at t = − √ 2. But in timeline, this value never exits as time always started at t = 0. See the following example: Solved Problem 1.21 A container is filled by water dripping from a tap. The height of water in container is a time function and it is given by h = t3 −6t+8. Find the maximum rate of water filling to the container. Solution The water filling function is h = t3 − 6t − 8 = f(t) This function gives f(−∞) = −∞ and f(∞) = ∞. Hence this function have global and may have local extrema. Function values at local extrema tell us whether water filling is maximum or minimum in definite time duration. So, to find the critical points of extrema, we have f ′ (t) = 0. So, 3t2 − 6 = 0 It gives two critical points, t = √ 2 or t = − √ 2. t f(t) f ( t ) = t 3 − 6 t − 8 b b f ′ (t) = 0 f ′ (t) = 0 (a) t f(t) f ( t ) = t 3 − 6 t − 8 b b f( √ 2) f(− √ 2) b b √ 2, 0 − √ 2, 0 (b) To check whether function is maxima or minima at a given critical point, f ′′ (t) 0 or f ′′ (t) 0 respectively. Now f ′′ (t) = 6t This is negative at t = − √ 2 and at this point filling function shall be maxi- mum. So filling is maximum at time t = − √ 2. This time never exists in time line, hence maximum water filling rate shall never achieved at local extrema.
  • 42. 30 Maxima Minima Solved Problem 1.22 A moving distance relation for a particle is y = t3 − 6 ∗ t2 + 11 ∗ t − 6. Find the extremum speed of the particle if possible. Solution The distance relation is y = t3 −6t2 + 11t−6. Velocity relation will be derivative of the distance relation y. So, v = d dt y = 3t2 − 12t + 11 This velocity curve has extrema points. To find these extrema points, we will get v ′ (t) = 0. So v ′ (t) = 6t − 12 = 0 This gives t = 2, There is only one extrema point, hence there will be either minima or maxima at this point. x y(x) v(x) y(x) b v ′ (x) = 0 (a) x y(x) v(x) y(x) b v(2) b (2, 0) (b) To check, there is maxima or minima, get v ′′ (t). v ′′ (t) = 6 0 This is positive value irrespective of t. So, there shall be minimum velocity at t = 2. The minimum velocity is v(2) = 3 × 22 − 12 × 2 + 11 = −1 This is desire result. Solved Problem 1.23 Find the dimensions of rectangle inscribed in semi- circle of radius r having maximum area. Solution Let there is semi-circle of radius r. A rectangle is inscribed in- side this semi-circle as shown in first part of below figure. Let the dimensions of the rectangle are 2x and y.
  • 43. 1.6. APPLIED PROBLEMS 31 1 1 −1 −2 x y b r x y 2 −2 1 −1 x A Now area of the rectangle is A = 2xy This is two variable relation. We can convert it to one variable relation if x and y are given in relation with r (constant). From the first part of above figure r2 = x2 + y2 Replacing y in the area relation, we have A = 2x √ r2 − x2 The area is maximum, where x gives maxima point on the area curve. To get critical points, put A ′ (x) = 0. A ′ (x) = 2x × 1 2 × (r2 − x2 )−1/2 × −2x + 2 √ r2 − x2 = 0 On solving it, we have x2 = r2 2 ⇒ ±x = r √ 2 ⇒ x = ∓ r √ 2 1 1 −1 −2 x y b r x y 1 1 −1 −2 x y √ 2 r r/ √ 2 To check whether area is maximum or minimum at x = ∓ r √ 2 , we have to find the sign of A ′′ (x) at the given x points. A ′′ (x) = − 6x √ r2 − x2 − 2x3 (r2 − x2) 3 2
  • 44. 32 Maxima Minima Value of A ′′ (x) at x = ∓ r √ 2 are A ′′ r √ 2 = − 6 × r √ 2 r r2 − r √ 2 2 − 2 × r √ 2 3 r2 − r √ 2 2 3 2 0 At this point area is maximum. Similarly, A ′′ − r √ 2 = − 6 × − r √ 2 r r2 − − r √ 2 2 − 2 × − r √ 2 3 r2 − − r √ 2 2 3 2 0 At this point area is minimum. Now, the maximum area of the inscribe rectangle is at x = r √ 2 . Now, the maximum area of inscribed rectangle is A = 2 × r √ 2 × s r2 − r √ 2 2 = r2 The dimensions of the rectangle are 2x = √ 2 r and y = √ r2 − x2 = r/ √ 2. Solved Problem 1.24 Cost of production of x articles is given by c(x) = 2x3 − 4x2 +4 and revenue received from the selling of same articles is r(x) = x2 −1. Find the production of articles when profit is maximum. Solution
  • 45. 1.6. APPLIED PROBLEMS 33 x p(x) c(x) = 2x3 − 4x2 + 4 r(x) = x2 − 1 b b b c ′ (x) = 0 c ′ (x) = 0 r ′ (x) = 0 (a) x p(x) c(x) = 2x3 − 4x2 + 4 r(x) = x2 − 1 b b b c(0) c(1.33) r(0) b b b (0, 0) (1.33, 0) (b) Figure 1.5: If cost of production of ‘x’ items is c(x) and revenue received from selling of ‘x’ articles is r(x) then profit of selling of ‘x’ items is p(x) = r(x) − c(x). If r(x), c(x) are differentiable for all x 0 then maximum profit occurs at p ′ (x) = 0. Now d p(x) dx = d r(x) dx − d c(x) dx At maximum profit d p(x) dx = 0 ie d r(x) dx = d c(x) dx Substituting the values of r(x) and c(x), we have 2x = 6x2 − 8x On solving it x = 0 or = 5 3 ≈ 1.666 The second derivatives of r(x) and c(x) at this value of ‘x’ are d2 r(x) dx2 = 2 = +ve
  • 46. 34 Maxima Minima and d2 c(x) dx2 = 12x − 8 = +ve respectively. The profit is maximum when production units are near about 1.666 in factor. Solved Problem 1.25 A cabinetmaker needs wood to produce 5 furnishings each day. He orders wood from woodseller periodically. The delivery cost of wood is Rs. 5000 per supply order. The storage cost of one furnished cabinet is Rs. 10 per day. Considering the delivery cost and storage charges, calculate the period after which he should order to minimize his average daily cost. Also calculate the quantity of wood to be delivered in one delivery. Solution x c(x) y = 5000 x y = 25 · x c(x) = 5000 x + 25 · x x-min Figure 1.6: Assume the cabinetmaker should order wood after ‘x’ days. In ‘x’ days he will produce total furnishings ‘5x’. The average cost of storage S(x) of furnished cabinet for ‘x’ days is product of furnished cabinets, number of days to be stored, cost of storage per unit per day and divided by two (for average). S(x) = 5x 2 · x · 10 Total cost T(x) of delivery and storage during two consecutive delivery is T(x) = 5000 + 5x 2 · x · 10
  • 47. 1.6. APPLIED PROBLEMS 35 Average daily cost C(x) during the two consecutive delivery is C(x) = 5000 x + 25x, x 0 When x → 0 or x → ∞, the average daily cost becomes large, hence the minimum value of ‘x’ should exists in between these two limits ie 0 x ∞. Now to get the critical point, differentiate cost relation with respect to ‘x’. d C(x) dx = − 5000 x2 + 25 On solving, two critical points are x = ± √ 200 ≈ ±14. Only x = √ 200 ≈ 14 lies in the domain of ‘x’. To verify the minimum cost, second derivative of average day cost at x = 14 is d2 C(x) dx2 14 = 10000 x3 14 = +ve This means, cabinet maker should ask for delivery of wood in period of 14 days. The average daily cost in rupees is C( √ 200) = 5000 √ 200 + 25 × √ 200 ≈ 707 The quantity of wood should schedule a delivery is Q = 5 · 14 = 70 units. Solved Problem 1.26 Prove that the maximum volume of a cone that is inscribed in a sphere of radius R is 8 27 times of the volume of sphere. Solution Let a right circular cone is constructed inside a sphere whose radius is R. Base radius of right circular cone is r and height is h. Semi- vertical angle of cone is θ (say).
  • 48. 36 Maxima Minima r h l l 2 R R b O A B C D E θ Figure 1.7: Inscribing a right circular cone in a sphere of radius R. Now volume of cone is given by V = 1 3 πr2 h (1.1) Both r and h are variables. Converting radius and height of cone if form of radius of sphere. From figure sin θ = l 2 R That gives l = 2R sin θ (1.2) Again sin θ = r l Here l is slant height of the right circular cone. cos θ = h l
  • 49. 1.6. APPLIED PROBLEMS 37 Substituting the values of r and l in equation (1.1) V = 1 3 πl2 sin2 θ l cos θ (1.3) Substituting the value of l in above relation from equation (1.2), volume of cone is V = 8 3 πR3 cos4 θ sin2 θ (1.4) To find the maximum volume differentiate to the equation (1.4) with respect to θ and equation it to zero. d dθ V = d dθ π 8 3 R3 cos4 θ sin2 θ Applying product rule for differentiation in right hand side of above equation. Hence d dθ V = π 8 3 R3 cos4 θ 2 sin θ cos θ − 4 cos3 θ sin θ sin2 θ = 0 On simplification 2 cos2 θ = 4 sin2 θ This gives tan θ = 1 √ 2 (1.5) Applying right angle triangle rule sin θ = 1 √ 3 And cos θ = √ 2 √ 3 Now for maximum volume second derivative test for volume of cone must be negative for the value of sin θ and cos θ. Now second derivative value of cone volume is d2 dθ2 V = d2 dθ2 π 8 3 R3 cos4 θ sin2 θ
  • 50. 38 Maxima Minima Second derivative is d2 dθ2 V = 8 3 πR3 12 cos2 θ sin4 θ − 22 cos4 θ sin2 θ + 2 cos6 θ Substituting the values of cos θ and sin θ in above relation d2 dθ2 V = 8 3 πR3 24 27 − 88 27 + 16 27 = −ve (1.6) This means volume of cone is maximum for given values of cos θ and sin θ. Now Substituting the values of cos θ and sin θ in relation (1.1) V = 8 3 πR3 4 9 1 3 V = 8 27 πR3 4 3 | {z } Volume of Sphere (1.7) This gives that maximum volume of cone is 8 27 times of the volume of sphere. Solved Problem 1.27 Find the maximum area of rectangle that is inscribed in ellipse x2 a2 + y2 a2 = 1. Solution A rectangle is drawn inside the ellipse as its four corners are at the ellipse. Assume a point A whose coordinate is (x, y). From symmetry other three coordinates of corners of rectangle are B(x, −y), C(−x, −y) and D(−x, y). Now length of side AB of rectangle is 2y and length of side BC is 2x. Area of rectangle is
  • 51. 1.6. APPLIED PROBLEMS 39 x y O A B C D (x, y) (x, −y) (−x, −y) (−x, y) b b b b 2y 2x Figure 1.8: Inscribing a rectangle in a ellipse. A = 2x × 2y (1.8) From the definition of ellipse x2 a2 + y2 a2 = 1 y = b a √ a2 − x2 (1.9) Now the area of rectangle is A = 4x b a √ a2 − x2 (1.10) For maximum or minimum area of rectangle, first derivative of equation (1.10) with respect to x would be make equal to zero to find the extrema points. ie dA dx = 4 · b a · √ a2 − x2 − x2 √ a2 − x2 For stationary values of x equating above relation to zero and solving for x. The result is a2 − x2 = x2 x = ± a √ 2
  • 52. 40 Maxima Minima If are of rectangle is extremum then second derivative test of area A should be negative d2 A dx2 = 4 · b a · − 3 x √ a2 − x2 − x3 (a2 − x2) 3 2 # (1.11) On substituting the value of x, d2A dx2 is negative, hence area of the rectangle inscribed inside the ellipse is maximum when x = ± a √ 2 Now maximum area of inscribed rectangle is A = 4 · b a · a √ 2 s a2 − a √ 2 2 (1.12) On solving equation (1.12), maximum area of rectangle is A = 2ab (1.13) Solved Problem 1.28 Find the maximum area of isosceles triangle that is inscribed in ellipse x2 a2 + y2 a2 = 1. Solution A isosceles triangle is drawn inside the ellipse as its tree vertices are at the ellipse. Assume a point A whose coordinate is (a, 0). From sym- metry of isosceles triangle other three coordinates are B(−x, −y), C(−x, y) and D(−x, 0). Now length of height AD of triangle is (x + a) and length of base BC of triangle is 2y. Area of triangle is
  • 53. 1.6. APPLIED PROBLEMS 41 x y b b b b O A B C D (a, 0) (−x, −y) (−x, y) (−x, 0) a + x 2y Figure 1.9: Inscribing a isosceles triangle in a ellipse. A = 1 2 2y × (x + a) (1.14) From the definition of ellipse x2 a2 + y2 a2 = 1 y = b a √ a2 − x2 (1.15) Now the area of triangle is A = (x + a) × b a √ a2 − x2 (1.16) To find maximum or minimum area of triangle, first derivative of equation (1.16) with respect to x would be make equal to zero for extrema points. ie dA dx = b a · √ a2 − x2 − x(x + a) √ a2 − x2 For stationary values of x equating above relation to zero and solving for x. The result is 2x2 + ax − a2 = 0 Solving it by using Shridharacharya formula (quadratic solution method) x = a 2 , −a
  • 54. 42 Maxima Minima If are of rectangle is extremum then second derivative test of area A should be negative d2 A dx2 = − x + a √ a2 − x2 − 2 x √ a2 − x2 − x2 (x + a) (a2 − x2) 3 2 (1.17) On substituting the value of x = a 2 , d2A dx2 is negative, hence area of the rectangle inscribed inside the ellipse is maximum when x = a 2 Here x = −a can not be considered as the value of y at this value of x is zero and it would give minimum area of triangle ie zero. Now maximum area of inscribed rectangle from equation (1.16) is A = a 2 + a × b a r a2 − a2 4 (1.18) On solving equation (1.18), maximum area of isosceles triangle is A = 3 √ 3 4 ab (1.19) Solved Problem 1.29 An open-top box is to be made by a small congruent square of a 2m × 2m sheet of tin and bending up the side. How large should the square cut from the corners so that it can hold maximum amount of contents. Solution 2 2 (a) Base 2 2 x x (b) 2 − 2x 2 − 2x x x x (c) x f(x) Maximum y = x(2 − 2x)2 0 ≤ x ≤ 1 (d) Figure 1.10: Fabrication of a Box. Figure (a) shows a square tin sheet, (b) shows the sheet cut out in its corner by a dimension of x × x, (c) a box is framed by bending up the cropped side and (d) the volume of box is graphed as function of x.
  • 55. 1.6. APPLIED PROBLEMS 43 A box is framed by cropping corners of side x × x from a tin sheet of size 2m × 2m and then bending the sides as shown in figure 1.11(c). The volume of box is V (x) = x(2 − 2x)2 From the figure, it is clear that the volume of box will be zero if x = 0 or x = 1 and domain of x is 0 ≤ x ≤ 1. volume of box will be non zero if 0 x 1 as shown in graph in figure 1.11(d). for maximum volume, derive the volume function with respect to ‘x’. d dx V (x) = d dx x(2 − 2x)2 On differentiation d dx V (x) = −4x(2 − 2x) + (2 − 2x)2 To get the zeros, putting right hand side equal to zero and on solving it gives value of x = 1/3. This value of ‘x’ lies in between the domain of ‘x’, hence the critical volume will be obtained at this point of ‘x’. Now critical volume is V(1 3 ) = 1 3 2 − 2 3 2 On solving it V = 0.592 m3 The maximum volume is 0.592 m3 and cutout squares should have side of 1/3m. Solved Problem 1.30 A box with lid is to be made by a small congruent rectangular thin sheet of size 3m × 2m. Find the dimensions so that it can hold maximum amount of contents. Solution
  • 56. 44 Maxima Minima 3 2 (a) Base Lid 3 2 x x x (b) (3 − 2x)/2 2 − 2x x x x (c) Figure 1.11: Fabrication of a Box. Figure (a) shows a rectangular tin sheet, (b) shows the sheet cut out in its corner and (c) a box is framed by bending up the cropped side. A box is framed by cropping of corners as shown in the figure 1.11(b). The volume of box is V (x) = (3 − 2x) 2 (2 − 2x)x From the figure, it is clear that the volume of box will be zero if x = 0, x = 1 or x = 1.5. But x = 1.5 i.e. 2x = 2 × 1.5m = 3m 2m is not possible in case of width of the tin sheet, hence domain of x is 0 ≤ x ≤ 1. volume of box will be non zero if 0 x 1 as shown in graph 0.5 1.0 1.5 x f(x) Maximum y = (3−2x) 2 (2 − 2x)x 0 ≤ x ≤ 1 For maximum volume, derive the volume function with respect to ‘x’. d dx V (x) = d dx (3 − 2x) 2 (2 − 2x)x On differentiation d dx V (x) = 12x2 − 20x + 6 To get the zeros, putting right hand side equal to zero and on solving it gives value of x = 0.392 and x = 1.274. Second value of x = 1.274 is outside the domain of x hence only x = 0.392 lies in between the domain of ‘x’, hence
  • 57. 1.6. APPLIED PROBLEMS 45 the critical volume will be obtained at this point of ‘x’. Now critical volume is V0.392 = (3 − 2 × 0.392) 2 × (2 − 2 × 0.392) × 0.392 On solving it V = 0.528 m3 The maximum volume is 0.528 m3 and height of box should be 0.392m. Solved Problem 1.31 An open channel pipe having cross section like a trape- zoid as shown in figure 1.12 (a). How are the trapezoidal sides slanted so that the volume of open channel pipe is maximum. A pump is delivering water one cubic meter per second when it work in its full capacity. Can the pump work on its full capacity? If not then find the working capacity of the pump so that the open channel pipe does not overflow when all the dimensions of the open channel pipe are of one meter. Assume that open channel pipe can intake and deliver water equivalent to its volume in one second. Solution θ h b (a) b b b b b b b b b b b b bcb l (b) Figure 1.12: Water flow in open channel pipe. The open channel pipe having cross-section of trapezoid shape is shown in figure (1.12) (a). The cross-section has base b and slant height h. The area of cross section is Ac = b × h cos θ + h cos θ × h sin θ The volume of open channel pipe is Voc = (b × h cos θ + h cos θ × h sin θ) × l (1.20)
  • 58. 46 Maxima Minima The parameters of open channel, h, b and l are constant, hence volume of channel depends on the value of θ. The volume of open channel will be zero if θ = 90o and bhl if θ = 0o . To get the maximum volume at any angle θ within the domain of angle 0o θ 90o , we first find the first derivative dVoc dθ = d dθ [(b × h cos θ + h cos θ × h sin θ) × l] (1.21) θ V V = h cos θ(b + h sin θ)l Figure 1.13: Volume of open channel is graphed as function of angle θ. dVoc dθ = −bhl sin θ + h2 l cos(2θ) At maximum or minimum value of volume of open channel, right hand side of above relation must be zero. Hence −bhl sin θ + h2 l cos(2θ) = 0 On simplification b sin θ = 2h cos θ sin θ Or cos θ = b 2h sin θ = 0 These relations gives values of angle θ = cos−1 b 2h (1.22) θ = 0
  • 59. 1.6. APPLIED PROBLEMS 47 Second value of θ is not within domain hence first value is acceptable. The volume of open channel will be maximum if second derivative of volume function is negative, now d2 Voc dθ2 = −bhl cos θ − 2h2 l sin(2θ) = −ve Now the maximum volume of open channel pipe from equation (1.20) is Voc =  b × h × b 2h + h × b 2h × h × s 1 − b 2h 2   × l Or Voc = b 4 × h 2b + √ 4h2 − b2 i × l This is maximum volume of the open channel pipe. Now, according to the question, all dimensions are of one meter hence volume of the open channel pipe is V = 0.933m3 The volume capacity of pump is 1m3 0.933m3 hence pump can not deliver water in its full capacity. Now to prevent the overflow of open channel pipe, pump should work at the capacity of Cpump = 0.933 1 × 100 = 93.3% of its full capacity. The water delivery is measured in per unit second. Solved Problem 1.32 A right angle triangle of hypotenuse k is translated about its one of the two mutually perpendicular sides to construct a right circular cone. Find the dimensions of the right angle triangle so that the volume of the cone is maximum. Solution h b (a) (b) Figure 1.14: Construction of right circular cone.
  • 60. 48 Maxima Minima The hypotenuse of constructing triangle is k and its height and base are h and b respectively. Now from the perimeter of right angle triangle as shown in figure (1.14) (a) k = √ h2 + b2 (1.23) The volume of right circular cone is V = 1 3 πb2 h (1.24) The hypoteneous of right angle triangle is constant k, The volume of right circular cone is zero if either of the base or the height is equal to zero or equal to k. Hence the domains of base and height for non zero volume of right circular cone will be 0 h k and 0 b k. Eliminating the base variable in volume function by using equation (1.23) V = 1 3 π(k2 − h2 )h (1.25) For maximum volume, the first derivative of volume function (1.26) should be zero dV dh = 1 3 π(k2 − 3h2 ) (1.26) θ V V = 1 3 π(k2 − h2 )h Figure 1.15: Volume of constructed cone is graphed as function of height h. At maximum or minimum value of volume of constructed cone, right hand side of above relation must be zero. Hence 1 3 π(k2 − 3h2 ) = 0 On simplification h = k √ 3 , 0 h k (1.27)
  • 61. 1.6. APPLIED PROBLEMS 49 This value of height is acceptable as it is within the domain of height and gives the value of base b = √ 2k/ √ 3 within the domain of base. The volume of constructed cone will be maximum if second derivative of volume function (1.26) is negative, now d2 V dh2 = 1 3 π(0 − 6h) = −ve Now the maximum volume of constructed cone from equation (1.26) is V = 1 3 π(k2 − k2 3 ) k √ 3 Or V = 2 9 √ 3 πk3 This is maximum volume of the constructed cone. Solved Problem 1.33 A both ends closed right circular cylindrical cane of 1lt is to be made from tin sheet. How the dimensions are specified so that the material is used is minimum. Solution (a) (b) x A min A = 2πr2 + 2000 r r 0 (d) Figure 1.16: Cylinder (a) this short and wide while cylinder (b) is thin and tall. Figure (c) is the area graphed as function of radius. From the figure, assume radius and height of the right circular cylindrical can are r and h respectively. The volume of the cylindrical can is 1000cm3 . Hence V = πr2 h = 1000
  • 62. 50 Maxima Minima If thickness of the tin sheet is not consider during the calculations then the term material means the total surface area of the can. Now the material is A = 2πr2 |{z} circular ends + 2πrh | {z } circular wall Eliminating height variables from surface area equation by using volume relation. A = 2πr2 + 2πr 1000 πr2 = 2πr2 + 2000 r If r = 0, the volume of right circular cylindrical can will be zero. Hence the domain of ‘r’ is r 0. Now differentiating the area term to get the optimum value of radius r. dA dr = 4πr − 2000 r2 0 = 4πr − 2000 r2 On simplification r = 3 r 500 π ≈ 5.42 The second derivative of area relation is dA dr = 4π + 4000 r3 and it is positive within domains of r 0. Hence at this value of ‘r’ the one liter can required minimum tin sheet. Now the dimensions are h = 1000 πr2 = 2 × 500 π × 1 r2 = 2 × r3 r2 = 2r Hence, one liter can has height equal to the diameter of the can. Radius height of the can should be r = 5.42cm and h = 10.84cm.
  • 63. 1.6. APPLIED PROBLEMS 51 Solved Problem 1.34 A cone is constructed from a flat circular tin sheet of radius r by removing a sector of angular arc θ. Find the height and value of θ so that volume of cone is maximum. Solution r (a) θ (b) (c) r (d) Figure 1.17: Figure (a) a circular sheet, (b) a wedge cut out from the circular sheet, (c) two ends at perimeter are joined together to shape it like a cone and (d) finally a framed cone. The radius of circular sheet is r, so its perimeter is 2πr. Now a section is removed from the circular sheet. The angular projection of removed section at the center of circular sheet is θ. Two ends at the perimeter circular sheet are placed in contact to fabricate a right circular cone. The minimum value of θ will be 0 and maximum value will be 2π. The volume of cone is zero at this minimum and maximum values of θ. For a right circular cone, the domain of θ will be 0 θ 2π. Remaining part of thin sheet after removing the section will form the circular base of right circular cone. Lateral height of cone is r. Now, the perimeter of original circular sheet is Pc = 2πr The removed section has arc length P′ c = rθ, 0 θ 2π Remaining part of sheet has arc length Pc − P′ c P = 2πr − rθ, 0 θ 2π The perimeter of the right circular cone is P P = 2πr − rθ, 0 θ 2π
  • 64. 52 Maxima Minima Radius of right circular cone is P/2π R = r − rθ 2π , 0 θ 2π The slant height of the right circular cone is r. The height of right circular cone is h = √ r2 − R2 Now the volume of the right circular cone is V = 1 3 πR2 h Substituting the values of R and h, volume of right circular cone becomes V = 1 3 π r − rθ 2π 2 × v u u t r2 − r − rθ 2π 2 # , 0 θ 2π On solving it V = 1 3 πr3 1 − θ 2π 2 × s θ π − θ2 4π2 , 0 θ 2π θ V (θ) Maximum V (θ) = 1 3 πr3 h 1 − θ 2π i2 × rh θ π − θ2 4π2 i 0 θ 2π Figure 1.18: Fabrication of a right circular cone. Volume of right circular cone is graphed as a function of wedge angle θ. For maximum volume, first derivative of the volume relation must be equal to zero. So, dV dθ = d dθ 1 3 πr3 1 − θ 2π 2 × s θ π − θ2 4π2 # = 0, 0 θ 2π
  • 65. 1.6. APPLIED PROBLEMS 53 Or 1 2 1 − θ 2π 2 1 π − θ 2π q θ π − x2 4π − 1 − θ 2π q θ π − θ2 4π π = 0 Using help of CAS software, θ has three values, θ = 0.93, θ = 4.49 and θ = 6.28. Only θ = 0.93 is fall under the domain of 0 θ 2π, hence the wedge angle must be equal to θ = 0.93 radian or θ ≈ 58.17o . Now the height of cone is h = s r2 − r − rθ 2π 2 = r s 1 − 1 − 0.93 6.28 2 ≈ 0.52 r Or h r ≈ 0.52 This is height to radius ratio.